Sunteți pe pagina 1din 104

CIVIL PROCEDURE SRPING 06 PROFESSOR PAGE Choosing the COURT where the trial will be tried.

d. Judicial Power of Courts given in Article III of the Constitution y federal court has power to adjudicate federal laws y federal dist court can be used for cases between citizens of different states Title 28 1332: District Courts shall have original jurisdiction of all civil actions arising under the Constitution, laws, or treaties of the United States Title 28 1332: District courts shall have original jurisdiction of all civil actions where the matter in controversy exceeds the sum or value of $75,000 between -citizens of different States -citizens of a State and citizens or subjects of a foreign state Federal Rule 4(k): Very important rule; will be an exam question Subject Matter Jurisdiction:The power or authority of a court to decide a particular type of legal dispute. For example a family court normally has jurisdiction over disputes involving members of a family and conversely lacks jurisdiction over disputes not involving members of the family. a. State trial courts with general jurisdiction are competent to hear all types of disputes not exclusively assigned to other state courts. State trial courts with limited jurisdiction have subject matter jurisdiction over matters expressly assigned to them. b. Federal Courts are limited jurisdiction b/c the Constitution permits them to decide only certain types of disputes. Hawkins v. Masters Farms P filed in Federal District Court in Kansas. Suit for wrongful death brought by representative of the estate. D wants to get the case dismissed for lack of subject matter jurisdiction. Ds dispute that there is a complete diversity among the parties, and want to dismiss for lack of subject matter jurisdiction. Case starts in Federal District Court. - chose to bring suit in Federal court, although could have brought it in state court. Proceeding to resolve a motion for lack of subject matter jurisdiction

Issue: Is there diversity in this case? Rule: Need to look at the citizenship of the deceased not the representative of the estate.

y Need complete diversity; each plaintiff is diverse from each defendant y Citizenship of a state depends on where you are domiciled domicile is established by physical presence in a place in connection with a certain state of mind concerning ones intent to remain there. (if you have an intent to remain somewhere temporarily then you will not be domiciled. has to be a indefinite intent to remain in the state) y Usually determine citizenship at the time the case was filed. Court looks to see if deceased had an intent to stay in Kansas. Holding: Plaintiffs case gets thrown out. Not an adjudication on the merits, just found no jurisdiction. Plaintiff has the option to try to refile their claim in state court. Notes: y Burden in on plaintiff to establish citizenship y If the case had been adjudicated then under the doctrine of res judicata a case cannot be retried. y Reasons why you would want to try case in Fed. instead of State court? -better judges -better juries -faster speed in Federal Court ISSUE: Citizenship of parties; whether they are citizens of different states. Look at 1391 on page 326. Personal Jurisdiction:Power of the court to decide the rights of the defendant. Once subject matter jurisdiction is established, the next step in selecting a proper court is to determine in which state the lawsuit (whether it is to be filed in state or federal court) can be brought. This consideration raised the issue of personal jurisdiction. a. Traditional way to assert personal jurisdiction is to serve process to the defendant in the state. b. Definition: Whereas subject matter jurisdiction is defined as judicial power over the subject matter of the lawsuit, personal jurisdiction is defined as judicial power over the person or property involved in the suit. c. There are two sets of rules governing personal jurisdiction: rules that apply to suits in federal court, and rules that apply to suites in state courts. - modern rule has expanded a state courts ability to exercise personal jurisdiction over defendants. Venue:Says within a court system that has subject matter jurisdiction which District or court is proper for the case to be tried in. The basic reason for this is convenience. Service of Process: Rule 4 establishes two alternative methods of notifying the defendant. The first, waiver of service, is informal and inexpensive: the Plaintiff mails

the D the complaint forms, and if the D returns the form than the action can continue. The second is a service summons, which is formal and expensive. Lawyers Responsibility: Lawyers must not bring frivolous claims that wasted valuable court time. Rule 11 enforces this responsibility by requiring lawyers to certify that, based upon a reasonable investigation of the facts and the controlling law, the complaint is not meant to harass the other party, and gives judges the authority to sanction lawyers for such violations. Bridges v. Diesel Service Inc.- RULE 11 Case arises under Federal Law so it is tried in Federal Court. Facts: P sued D claiming that D violated her rights under the Disabilities Act when D dismissed P from her job on account of Ps disability. Because P failed to file a charge of discrimination with the EEOC before commencing action, the District Court dismissed Ps action for failure to exhaust administrative remedies. D now moves for sanctions pursuant to Rule 11 Issue: Should monetary sanctions in the form of attorneys fees be imposed on Ps counsel for failure to exhaust administrative remedies? Holding: The court does not impose sanctions on the lawyer. The primary goal of Rule 11 sanctions is to deter improper conduct, not to shift attorneys fees. Ps counsel immediately acknowleged the error and attempted to rectify the situation. Rule 11 sanctions should be reserved for exceptional circumstances in which the claim asserted is unmeritorious of frivolous. Notes: Safe Harbor Provision The other party has to give the party in error notice of their mistake and give them 21 days to reply. Bell v. Novick Transfer Facts: P was injured in an accident with D. Ps tort action was removed (1441) at Ds request from state court to Federal Court, thereby placing the case under the authority of the Fed. Rules. Ds moved to dismiss the action on 2 grounds: (1) that the complaint failed to state a claim where relief can be granted (Rule 12) and that the allegations in the complaint lacked specificity (Rule 12(e)) action based on diversity Issue: Were the allegations in Ps complaint sufficient under Rule 8? Held: Yes. Both motions overruled.

y The allegations of Ps complaint were specific enough to inform Ds of the nature of Ps legal theory and hence were sufficient to enable Ds to frame responsive pleading y Rule 8 which sets forth the basic premise for federal pleadings, requires only a short and plain statement of the claim showing that the pleader is entitled to relief. Because fed. procedure is more liberal than state procedure, Ps allegations may not be sufficient under Maryland practice. Federal Rules deemphasize pleading, and accentuate discovery.

Notes: In a diversity case you provide the substantive law of the state that the court is sitting, but the federal rules of procedure still do apply. y Pleading for negligence cases look at FORM 9 (pg.172) y If there is not sufficient evidence to support a case, either party can ask for summary judgment. Defenses (RULE 12) (every kind of defense can be used in the answer) (only lack of authority and failure to state a claim can be brought up in a motion) Defendants use defenses to beat a claim y List of defenses in 12(b) 1. Lack of authority to decide: - lack of subject matter jurisdiction - lack of personal jurisdiction - improper venue - wrong serving process 2. Failure to state a claim (look at 4 corners of doctrine) - challenges the legal sufficiency of the claim; asks for something the law doesnt allow relief for - claim is to vague - in common law called a demurer 3. Denial (raises issue of fact which requires discovery to absolve) - in common law called traverse 4. Affirmative defense (Rule 8(c) ) - contributory negligence, statutory negligence, Assump. of risk, etc. - common law confession and avoidance y Certain defenses (not subject matter jurisdiction, not failure to state a claim) can be waived if certain procedures are not followed.

y In an answer a counter-point can be raised. Rule 13 allows for counterclaim and cross claim. permissive counterclaim: any claim in which D has against opposing party Say there are two defendants and one D has a claim against the other D, they will cross-claim. Goes in the answer 42(b) tries to ease the confusion by allowing for separate trials for each claim if necessary.

y Joinder provisions are lax in the federal rules. Bridgeport Music Inc. v. 11C Music In severance the court creates different actions, this case they severed the cases. This allows there to be a different judgment for each separate case. When the cases are not severed and rather there are two defendants, there will be the same holding for each. Facts: Ps engaged in publishing, recording, and distributing music. They sued over 770 music companies for copyright infringement. Case is in Federal Court because it is a violation of Federal Laws. Ps complaint includes 486 counts most of which contain multiple claims and is 901 pages in length, not including exhibits. D argues that Ps complaint fails to give a short and plain statement of the claims required by FR 8, and also that Ps improperly joined Ds in violation of FR 20(permissive joinder). Issues: (i) Did Ps violate rule 20 by not properly joining Ds? Held: Yes. Ds motions for severance are granted. As a result of first holding, short and plain statement issue is moot y FR 20 has 2 basic requirements for joining party defendants: transactional relatedness and common question. To satisfy the transactional test the defendants to be joined must have rights asserted against them that arise from related activities. y Court finds that the sampling was not part of a single series of transactions or occurrences but rather many distinct instances of copyright infringement. y Court says that even if transactional test was satisfied they still would have severed cases to avoid causing unreasonable prejudice and expense to Ds and avoid a great inconvenience to the administration of justice. Notes: The transactional test is highly subjective. Other joinder provisions Rule 20-permissive joinder: What the plaintiff can do in the plaintiffs discretion. The plaintiff doesnt have to join all the claims it has, or does it have to include all transactions or occurrences the can be included in the lawsuit.

Rule 19 - Compulsory joinder: Defendant says that there should be another defendant added to this lawsuit. 19(a) talks about necessary parties; 19(b) talks about dispensable parties. y joint tortfeasors situation is not grounds for a necessary joinder y 19(b) the court decides that there is no way that they can render a judgment or proceed without joining a third party, then they must bring the other party in, or the case will be dismissed. Rule 14- allows D to assert a claim against someone who is not a party. If the P wins against D, then third party will have to pay for judgment (indemnity). D cant just say I am not liable, instead he will get paid back by third party Rule 24 -Intervention: Case where 3rd party wants to invite himself in, where the other parties dont want to. y intervention of right- party has right to join y permission intervention- allowed to intervene Interpleader: a device that allows someone who has a money, property, etc. who has conflicting claims with it to go to the court and allow them to decide what to do with it. Class actions: allows disputes in which you have large # of plaintiff who have fairly small individual claims that can be aggravated and is tried by a few parties that are the representatives to the class. y when the $ for the class action goes unclaimed, the court will give it cy pre which means they give it to schools, education, charities. y Recently we have had the class action fairness act- greatly relaxes subject matter jurisdiction for class actions. Can get class actions in Federal Court. PLEADING 1. complaint 2. motion to dismiss 3. answer 4. DISCOVERY Once we get passed pleading stage, the case cannot be dismissed on the pleadings. In discovery you are trying to get additional information that you have not obtained through the informal gathering of information. Rule 26 (a)(1) certain types of evidence has to be turned over automatically to other side, even with other side not asking for it. Must turn over witness list, and all documents. Rule 11 assumes that there will be some fact investigation without discovery. A lawyer has the responsibility to do some discovery without a court order compulsion.

What is within the scope of discovery? Depositions: Rule 30 and 45 are ones in which a third party can be subpoenaed to testify or produce documents.

3 Limitations to Discovery FR 26(b)(1)(2) 1. relevance (the material has to be able to make facts more or less likely to be discovered. information) 2. burdensome (cost/benefit analysis. if cost to party greatly outweighs the benefit of discovery it will not be allowed. information sought cannot be unreasonably cumulative or duplicative or can be obtained from another source with less burden or expense.) 3. privileged(privileged material cannot be discovered)

Butler v. Rigby Issue: (i) Is a listing of the total number of patients referred to AMG and MHC by Ps lawyers or firm discoverable as a matter of law under FR 26? (ii) is a computer printout of AMGs and HMCs current patients discoverable as a matter of law under FR 26? Held: (i) yes (ii) no y the discovery of the patient list is reasonably calculated to lead to evidence showing that Ps medical providers stood to profit form Ps court victory, and falls within the scope of FR 26. Although AMG and MHC have demonstrated that the expense of producing such a listing would be great, the burden does not outweigh the likely benefits. y Rule 501 of FR of Evidence provides that state privilege law should govern diversity cases. LA courts have broadly interpreted the scope of health care provider-patient privilege to include patient identity. Therefore this information is not discoverable. In diversity cases you apply substantive law of state, but Federal Procedural Laws!!! Class notes: Assuming this information is relevant we are still not allowing it to be discovered because the doctor-patient privilege is more important than the discovery process. If discovery were allowed it would jeopardize communication in the doctorpatient relationship. SUMMARY JUDGEMENT: (Fed Rule 56) After factual development of the case through discovery, enough information may be available to enable a lawyer to decide whether the case can be terminated prior to trial. If the pleadings, depositions, answers to interrogatories, and admission on file, together with the affidavits, show that there is no genuine issue as to any material fact, or whether

any reasonable jury can find for the non-moving party. Good way to think about summary judgment is to ask, what would happen with the trial? y Difference between a motion to dismiss for a failure to state a claim is that in dismissal the court would look at the claim in the complaint.If you have to consider anything else outside the complaint, you are no longer dealing with a motion to dismiss, you are looking at summary judgment. Houchens v. American Home Assurance Co. Facts: Houchens (P) filed a complaint for breach of contract against American Home (D) involving two life insurance policies issued by D covering Ps husband. Both policies required that Ps husband death be caused by accident. Husband takes trip to Thailand and is never heard from again. Several years passes and in Virginia he is presumed to be legally dead. D contended that there was no evidence of Ps husbands death or of accidental death. D was granted summary judgment. P appeals. Issue: Did the court err in granting the summary judgment for D? Held: No, P did not meet the burden of proving that her husband died accidentally. (Virginia substantive law puts burden on the plaintiff to prove this) y FR 56 mandates the entry of summary judgment, after adequate time for discovery and upon motion, against a party who fails to make a showing sufficient to establish the existence of an element essential to that partys case, and on which that party will bear the burden of proof at trial. y The appellate court must reverse a grant of summary judgment where it appears from the record that there is an unresolved issue of material fact, but, you cannot pile on inferences on inference. Class Notes: In this case, what would happen with the trial if summary judgment is not given? y Case would be docket for trial and then there would be jury selection. Have to find a jury that is a reasonable cross-section and not biased. Once jury is formed than trial is called. y Next would be the opening statement by both parties. P would put on the case, and then there would be cross-examination by D. P would try and prove that husband disappeared in Thailand and the concern would be to the defendant that the jury would be sympathetic to the P. y Then P would rest. D could then move for a directed verdict. In a directed verdict the motion says assuming the truths of the plaintiffs evidence, the plaintiff has no legal rights to seek relief; the facts have no legal significance. If directed verdict was denied, D would move on to their case. y D would present their case, and after D is done, both parties can move for a directed verdict. If no directed verdict, they move onto closing arguments.

y After closing arguments the jury can make a ruling, and the losing party can request a JNOV. Other Notes: In general if you have testimony for an essential fact of the case, even if its not very credible testimony, that is a situation in which you allow a jury to try the case. Norton v. Snapper Power Equipment Facts: P sued Snapper (D) for injuries while riding a Snapper lawnmower. At the close of Ps case and again at the close of all evidence, D moved for a directed verdict. This motin was denied, and was followed by a jury verdict for P, which was nullified by a judgment for JNOV in Ds favored. The judgment for JNOV was entered b/c the trial court believed that the evidence was insufficient to conclude that the equipment was defective. P appeals. Issue: (i) Was the evidence sufficient for a jury to reasonably conclude that the lawn mower was defective (ii) did the defect cause the injury? Held: Yes. Judgment reversed; case remanded. y Although the causation evidence in this case was circumstantial, the jury could reasonably have reached its verdict on the following bases: that when P applied the brakes, and effective dead mans device could have stopped the blades in as little as .7 seconds experts testified that given the amount of time that mower would have taken to slide six feet and given Ps testimony that both of his hands were on the handle bars until the mower hit the creek, a two or three second difference in blade stopping time would have avoided the injury.

y Enough evidence in which a reasonable jury could have inferred that the defect in the mower was the cause of the injury. y After the decision on appeal, the jury verdict is reinstated. Preclusive Effect that Judgment Has on Other Cases 1. Res Judicata: Says that if the plaintiff sues the defendant, the losing party, whether the P or D, may not sue the prevailing party in a 2nd lawsuit involving the same claim if the first lawsuit was decided on the merits. Never applies to people who are not involved in the suit. 2. Collateral Estoppel (issue preclusion): Provides that if the plaintiff or the defendant litigates an issue, the party against whom the issue is decided may not seek to relitigate that issue in a subsequent lawsuit. The issue of collateral

estoppel arises when the 2nd lawsuit involves a different claim. (In Rush it means that the city is estopped from claiming that they are not negligent in a 2nd suit).

3. Stare decisis:

Rush v. City of Maple Heights Facts: P was injured in a motorcycle accident when she struck a hole in the road. She sued the city (D) in municipal court, and won ($100) damages for the destruction to her property, as the court found the city negligent in the upkeep of the roads. P brings a 2nd lawsuit against D for damages to her person resulting from the accident. The trial court allows the P to keep the same negligence judgment in the 2nd action and P is awarded $12,000.D argues that that P is splitting the cause of action. Issue: Did Ps motorcycle accident give rise to 2 different claims (damage to property and damage to person), such that the lower courts were correct to apply the doctrine of collateral estoppel rather than the doctrine of res judicata in the second lawsuit. *** City says that you cant use collateral estoppel if there is only one event. P wants to rely on collateral estoppel (to show that city is negligent in both cases) and city wants to rely on res judicata (to say that the claim had already been brought in the first suit)*** Held: The weight of authority is that if Ds wrongful act is single, the cause of action, or claim, must be single y In this case, Ds act in not repairing the road was a single act therefore P will only have a single claim y Since P has a single claim the lower courts should have applied the doctrine of res judicata, which means that Ps lawsuit should have been dismissed. Res judicata comes into play when the second lawsuit seeks to relitigate the same claim litigated in the prior lawsuit, and bars such relitigation. Collateral estoppel comes into play when the second lawsuit addresses a different claim, and it precludes relitigation of issues decided in the first lawsuit. Notes: Why would you say that personal/property damages are separate? different types of evidence would be needed to prove personal and property damage security of your body is a totally different right then the security of your property

10

Scope of a cause of action (jurisdiction) called same transaction or occurrence (similar to the joinder criteria)

APPELLATE SYSTEM Most states (some still dont have intermediary appellate courts): Circuit Courts District Courts State Supreme Court y only state issues that are heard by the Supreme Court come from State Supreme court and have to do with Constitutional questions. Sup. Court wont disturb parts of ruling that deal with state law, only Constitutional matters. Federal System: District Courts (limited jurisdiction) Court Circuit Ct. of Appeals Supreme

Because of the Eerie Doctrine the Federal Courts will use the state law. If there is a blank area of the law, a Fed. Circuit court can certify a question to the state Supreme Court and get an answer on how they would rule on the law. What Decisions are Appealable? Apex Hosiery Co. v. Leader (Rule 34-discovery and production of documents) On review the court found that the discovery motion resembled others made before or during a trial to secure or to suppress evidence, such as applications to suppress a deposition, to compel the production of books or documents, for leave to make physical examination of a plaintiff, or for a subpoena duces tecum. The court held that orders made upon such application, so far as they affected the rights only of parties to the litigation, were interlocutory. The court noted that it was only when disobedience happened to result in an order that punished criminally for contempt that a party might have review by appellate proceedings before the final judgment in the case. The court held that the order was interlocutory was therefore not subject to appeal. Title 28 1291: Final Decisions of District Courts The courts of appeals (other than US Court of Appeals for Fed. Circuit) shall have jurisdiction of appeals from all final decisions of the district courts of the United States. Interlocutory appeals are statutory and judicial exceptions to the final judgment rule. The common purpose of the exceptions is to provide immediate appellate review of district court decisions affecting the substantial rights of the parties. y An order may not be immediately appealable, but if the party preserves the objection to that issue it might be used in the final decision. Final order doctrine: The general rule is that a District courts judgment must be final in order to be appealable. This important rule is found in 28 U.S.C. 1291 and is designed to avoid inefficiencies that result from granted piecemeal review (granting of summary judgment is a final order; denial or summary judgment is not a final order)

11

y final order doctrine prevents the possibility that there will be unnecessary delay in the trial, the possibility that the issue for appeal may turn out ot be trivial at the trials conclusion, and the possibility the case may be settled. STANDARD OF REVIEW 1. Jury trial: is there substantial evidence to overturn the verdict; the reasonable jury standard. Can a reasonable jury reach that result. 2. Bench trial: - rule 52; on finding of fact, review only if there is any clearly erroneous judgment. Conclusions of law are reviewed de novo

END OF OVERVIEW
in personam jurisdiction: power to adjudicate rights between a natural person, corporation, or unincorporated society. in rem: judicial power concerning property. its the power of the court to decide who the property belongs to. power to adjudicate the rights of the whole world to that piece of property. it has to attach the property out at the outset of the case.

Pennoyer v. Neff Facts: There was 3 judicial proceedings in this case. The first service was in personam. Mitchell, an attorney, sued Neff (P) in Oregon state court to recover attorneys fees P owed Mitchell. P did not make a general appearance in the action, was not a resident of Oregon, and was not physically present in the state at the time of service. Service of process was made by publication of the summons in a local newspaper; P failed to respond to the complaint and, consequently, a default judgment was entered in Mitchells favor. The second proceeding was quasi in rem. Mitchell had a judgment (piece of paper) but no money; he therefore obtained a writ of execution ordering Oregon sheriff to sell Ps property located in Oregon to pay off Ps debt to Mitchell. Pennoyer (D) obtained title to Ps property by purchasing the property from the sheriff. The final proceeding is in rem. P sued D in federal court (diversity in jurisdiction), claiming that P is the rightful owner of the property. The federal trial court ruled in favor of P, holding that the affidavits (sufficiency of who signed the affidavit to publish the summons) in the quasi in rem proceeding were defective. D takes his appeal to the Supreme Court. Issue: Did the Oregon state court have valid in personam jurisdiction over P in the first Mitchell v. Neff proceedings? Held: No, judgment affirmed on other grounds. y The common law rule for in personam jurisdiction has two requirements (i) all defendants, whether residents or non-residents, must be served with process personally; and (ii) all nonresident defendants must be physically present within the forum state when such service is made.

12

-The purpose of the first requirement is to give defendants an opportunity to respond. The purpose of the second requirement is to respect the borders of sister states: a state has power over persons or things within its territory and no power over persons or things outside its territory. y in personam jurisdiction: the power of a court to adjudicate the rights of the parties; have the right to adjudicate the rights of the defendant from any claim y in rem jurisdiction: a proceeding against property; pure form of in rem jurisdiction is to decide who owns a piece of property. 1. Quasi in remsubstantially in rem: Court acquires jurisdiction on the basis of having property in the state. Could only adjudicate the rights up to the amount that the property is worth. 2. (hypo) suppose the property was owned by Neff and attached properly at the beginning of the case. Neff is still elsewhere... Just the presence of the property in the state allows the State Court to adjudicate the claim. The court is using the property not to adjudicate who owns it, but use the property in the state as a basis to adjudicating the rights of the claim. 3. in quasi in rem - attachment - garnishment (sequestration) y Notice is a key element of in personam jurisdiction. This requirement is not met because Mitchell served Neff by publication which is inadequate notice. y Power is the other key element in in personam jurisdiction. The second requirement was not satisfied because P was a nonresident and was not physically within the sate at the time of service of process. Boundaries of the state set the absolute boundaries for that state to assert jurisdiction. - just being in the boundaries of the state, gives the state enough power to serve process *** In this case you needed to have both for jurisdiction, but they had neither. y If the property had been attached, the notice of publication would have been sufficient, because he would have had notice when he checked up on his property or the caretaker of the property let him know. y No in personam jurisdiction b/c he was not served personally within the boundaries of the state, no in rem b/c the property was not attached at the beginning of the proceedings. 2 Principles: States have this power because they have sovereignty 1. every State possesses exclusive jurisdiction and sovereignty over persons and property within its territory 2. no State can exercise direct jurisdiction and authority over persons or property not in their territory.

13

What says that States have to obey judgment of other states? Full Faith and Credit Clause Do States still have to give full faith and credit if the other state never obtained jurisdiction? y If in the original case the court lacked jurisdiction it can be collaterally attacked by the other state. Due Process Clause: says that as a matter of due process you have to have in personam and in rem jurisdiction. Notes on Case: In an appeal from a trial court decision, the court of appeals will affirm if it is the correct decision even if the appellate court disagrees with the specific ground for affirming the judgment. Some state laws says that if you want to conduct business in the state, you HAVE to appoint an agent for a service of process in that state; if you fail to do so they will serve a public officer of the state. (idea of consent) Implied Consent: example of man driving through state and getting into an accident. He can be served.

Examples of this idea (pg. 68) A from Minnesota B from North Dakota where B owns some land A sues B in Minnesota a) A serves B in ND; B moves to dismiss the Minnesota action for lack of personal jurisdiction y service of process on B is ineffective under the Pennoyer framework since court doesnt have power outside its jurisdiction y special appearance exception if you come to court just to raise the personal jurisdiction issue, you will not fall into the jurisdiction of the court. Under Federal Rules there is no special appearance. b) Same as (a) except that instead of appearing to object to the Minn. courts jurisdiction, B defaults and the court enters judgment against her. A then takes that judgment to ND to enforce; B appears in the enforcement proceeding and argues that the judgment is invalid and should not be enforced. j can apply the full faith and credit clause; North Dakota doesnt have to recognize judgment if there is no personal jurisdiction; can serve process if its out of state.

14

c) A serves B while B is traveling on business in Minnesota; B moves to dismiss for lack of personal jurisdiction j once the person is in the state, they are within the power of the state for them to be served with personal jurisdiction d) (NEED QUESTION) ND court would be bound by the law of Pennoyer and have to conclude that B had been subject ot personal jurisdiction on these circumstances e) (NEED QUESTION) Court in Pennoyer recognized the concept that as long as the property is physically located in the state, the Court has the power to adjudicate any rights for any claim made in the state up to the price of the land being sold. f) (NEED Question) Personal service is required g) h) i) If you dont file any motion for jurisdiction, and just raise all your defenses in the answer; but if you dont raise jurisdiction in answer, then it is to late. Jurisdiction is waived. Pg 75. Questions a) If you want to raise any of the defenses in 12(b), 12(g) says you have to raise them all at once b) no, not permissible, if you filed a motion, you had to include it in the motion, cant put it in the answer after there has been a motion. c) if you want to raise a lack of jurisdiction defense, you have to include it in the FIRST answer/motion you make

Side Case Milliken v. Meyer Facts: Two partners in an oil well sued eachother. Meyer, the D, had been a resident of Wyoming, but at the time of the suit served personally in Colorado. Meyer did not appear in Wyoming, where the court rendered judgment against him. Meyer attacked the Wyoming judgment collaterally. The U.S. Supreme Court held that the Wyoming judgment was valid. Holding: Domicile in a state is alone sufficient to bring an absent D within the reach of the states jurisdiction for purposes of a personal judgment by means of appropriate substitute service. Substitute service, [service other than personal service in the forum

15

state] has been uniformly upheld where the absent D was served at his usual place of abode in the state as well as where he was personally served outside the state. The authority of a state over one of its citizens is not terminated by the mere fact of his absence from the state. The state which accords him privileges and affords protection to him and his property by virtue of his domicile may also exact reciprocal duties. RECIPRICOL DUTIES

MODERN CONSTITUTIONAL FORMULATION OF POWER Introduction: Whether in personam, in rem, or quasi in rem, the exercise of valid state court jurisdiction today must comport with the Due Process Clause of the 14th Amendment, which means it must comport with fair play and substantial justice. Redefining Constitutional Power: A state statute must first authorize a court to exercise power over the D before one can even attempt to deal with the constitutional issue. The sate jurisdictional statute called a long arm statute. Thus if it is determined that the facts of the case fit within the four corners of the states long arm statute (i.e. that the statute reaches the case under consideration), then and only then, it must be determined whether the assertion of state power over the defendant pursuant to the statute is constitutional Whenever you have a personal jurisdiction question must look at the GRID for the contacts the defendant has within the state.

International Shoe v. Washington (overrule territorial jurisdiction for in personam jurisdiction) Issues: Is it permissible under the 14th amendment for a Washington court to exercise in personam jurisdiction over D on the basis of Ds activities within the state, where Ps lawsuit is limited to the recovery of unpaid contributions to the state unemployment compensation fund? Held: Yes. Judgment affirmed j The 14th amendments DUE PROCESS CLAUSE gives a court the authority to exercise in personam jurisdiction only if minimum contacts, ties or relations exist between the defendant and the forum state. j Such minimum contacts exist is this case because Ds in-state activities were neither irregular nor casual. They were systematic and continuous throughout the years in question and accorded D the benefits and protection of the laws of the state, including the right to resort to the courts for the enforcement of its rights

16

j D was doing business in Washington, was present in Washington on a continuous basis, and because fo that the exercise of in personam jurisdiction was fair it did not offend traditional notions of fair play and substantial justice Notion of fair play and substantial justice take into account - the burden on the defendant - the forum states interest in adjudicating the dispute - the interstate judicial systems interest in obtaining the most efficient result - share interest of the several states in furthering fundamental substantive social policies. *** MINIMUM CONTACTS IS THE TEST FOR THE DUE PROCESS CLAUSE FOR THE 14TH AMENDMENT OF THE CONSTITUTION. ITS THE FEDERAL TEST*** THIS IS THE GRID YOU LOOK AT FROM INTERNATIONAL SHOETO DETERMINE IF THERE WILL BE JURISDICTION IN THE CASE!!!!!!

RELATED

UNRELATED

Isolated & Casual Continuous & Systematic

MAYBE(Hess)

NO

YES (Intl Shoe)

MAYBE(Miliken)

j In Shoe the court puts the actions in the Continuous and Systematic/Related category therefore giving Washington jurisdiction over the matter. General v. Specific Jurisdiction General Jurisdiction: In some cases the D will have such substantial contacts with the forum state to make it fair to assert jurisdiction even over claims unrelated to those contacts. For example, General Motors has its chief place of business in Michigan. Under the theory of general jurisdiction, General Motors could be sued in Michigan even over contracts or torts concluded or committed in Idaho or Germany. Specific Jurisdiction: Where the Ds activities fall short of general jurisdiction, the minimum contacts test in International Shoe becomes important. In those caeses courts worry about the extent of those contacts and the relation between those contacts and the

17

claim on which the plaintiff is suing. Suppose driver has never been in Missouri but is so on the course of a vacation. Specific jurisdiction would allow an injured pedestrian to sue driver on a claim arising from that accident.

NOTES: State of Fla. is very aggressive in asserting general jurisdiction. Fla. asserted general jurisdiction over a plane crash that happened from a corporation from Belize, but since the corporation used Florida ports, airports, etc. Fla. could assert jurisdiction because they were doing business in the state. HYPOS ON 83: a) yes, specific jurisdiction: first look at contact [driving the truck through the state and negligent act by driver] Single act which is isolated and casual, and gives rise to the claim. b) no, no jurisdiction: the only contact Intl shoe with WY is that they drive through the state (isolated and casual). Here the claim for wrongful discharge is unrelated to activities in the state. Isolated & Casual +Unrelated = no jurisdiction. c) Missouri would have jurisdiction: the level of activity that takes place in the state is continuous and systematic and one that gave rise to the claim. Continuous and Systematic + Related = Jurisdiciton d) A corporation that is incorporated in a state is usually subject to that states jurisdiction (Delaware) LONG ARM STATUTES The first question before you get the constitutional question: 1. Is there a statute that allows the court to assert jurisdiction in these circumstance? ; then 2. if there is, he court must ask whether it would be constitutional under the Due Process Clause. The long arm statute authorizes the state courts to exercise jurisdiction over defendants based on specific types of contact with the forum state. Situations where it is totally constitutional to assert jurisdiction nation-wide, but if they want to assert jurisdiction over other countries, they use the minimum contacts test. Some states have more narrow jurisdictional standards that the Federal Standard. California test on pg 161: If you were to get a test question on the Cali. Statute you would have to acknowledge that you have to satisfy the long-arm statute (which is the same as the Constitutional/Federal test) and then do the Federal test even though they are both the same for all intents and purposes.

18

Gibbons v. Brown Facts: Mrs. Brown (P) sued Mrs. Gibbons (D) a Tex. resident, in Fla. for injuries arising from an automobile accident that took place in Montreal, Canada. D and Mr. and Mrs. Brown were riding together in the car. D gave the driver, Mr. Brown faulty directions, which led him the wrong way on a one-way street. A head-on collision resulted. P and D were injured. Two lawsuits were subsequently filed. In the first lawsuit, D sued Mr. Brown in Fla. Two year later, P, who was not a party to the first lawsuit, sued D in Fla. under the theory that D subjected herself to jurisdiction by bringing the prior lawsuit. Floridas long arm statute gives its courts jurisdiction over a defendant who is engaged in substantial and not isolated activity within the state. Issue: Does Fla. have in personam jurisdiction over D under its long-arm statute Held: No complaint dismissed

Application: First issue of whether Fla. has jurisdiction is the 1st prong, the Fla. long-arm statute, is satisfied. Dont get passed the first test because the statute is not satisfied. The court classifies the Fla. Long-arm statute is more specific. Requires more activities and contacts than the Constitution. Subsection (1) is the specific limitation section. Only claims arising from that action are allowed. Just because you own real property in a state, doesnt mean that you can subject someone to jurisdiction if its not related to the activity. Subsection (2) is the general jurisdiction provision.

j Even if we assume, without deciding, that bringing a lawsuit in the sate constitutes a substantial and not isolated activity under Flas long-arm statute, P has not shown that D has engaged in any relevant activity. The only activity that D has engaged in is defending in the present lawsuit. j Bringing a prior lawsuit in a state should not hang indefinitely over that person. Once you file a lawsuit and its over and you leave the state, you dont to subject that person to jurisdiction forever. You dont want to have it hanging over someone that just because they sued once in a state court, that they could be subject to be sued there afterward.

HOW STRANGE THINGS GOT UNDER PENNOYER

19

H (NC)

owed B (NC) $

owed E (MD) $

H went from North Carolina to Maryland, where E found out he was there and served him with process. H had to pay E in Maryland, and when H came back to North Car. B sued him for money also. H claimed res judicata, but MD court did not allow him to raise this defense. Supreme Court overturned. McGee v. Intl Life Insurance Facts: The beneficiary (P) of the decedents life insurance policy sued out of state insurance company (D) for non payment. D refused to pay P on the ground that the decedent committed suicide, a policy exclusion. Ds only contact with the forum state were the premium notices it mailed to the decedent, a resident in the forum state. The state court exercised jurisdiction. D appeals. Issue: Can the forum state exercise personal jurisdiction over D on a claim related to the insurance company? Held: Yes, judgment affirmed. j It is sufficient for purposes of due process that the suit was based on a contract that had substantial connection with the state Comments: This case illustrates the extensive reach of personal jurisdiction based on claims that relate to the defendants in-state activity (specific jurisdiction), in this case the premium notices D mailed to decedent. McGee gave International Shoe long arms. Shoe greatly expanded jurisdiction with the explosion of commerce; held that even a single contract that took place in a state, is enough to assert personal jurisdiction

Hanson v. Denckla Issue: Was Floridas exercise of in personam jurisdiction over the Delaware trust company constitutionally valid? Held: No. Judgment reversed. j Fla.s exercise of in personam jurisdiction over the DE trust company was constitutionally invalid because the trust company had no purposeful contacts with Florida. The in-state acts on the trustees part were not purposeful in the sense that the trustee did not purposefully avail itself of the privilege of conducting activities within the forum state, thus invoking the benefits and protections of its laws.

20

j There is a contact with the state that are factually caused by things that the trustee did, but what is missing is the purposefully availment. j Purposeful Availment emphasizes that the D must have made a deliberate choice to relate to the state in some meaningful way before she can be made to bear the burden of defending there. Unilateral contacts with the state will not due. j Hanson is the first SCOTUS case after Shoe to suggest that minimum contacts essetntially means purposeful act. Notes: unilateral actions by P, were not sufficient to establish the purposeful relationship the court said was necessary to establish minimum contacts. Difference between McGee and Denkla is that Denkla court limits McGee and says that defendant must have related activity in state; just any contact is not going to be enough, has to have purposeful availment.

Shaffer v. Heitner (overrule territorial jurisdiction for in rem jurisdiction; extends International Shoe to quasi in rem jurisdiction) Facts: Heitner (P) a nonresident of Delaware, filed a shareholders derivative suit in Delaware against Greyhound Corp, a Delaware corporation with its principal place of business in Arizona. P, a shareholder in Greyhound is suing the 28 present and former officers and directors of the company (Ds). Ds alleged wrongful act took place in Oregon. A Delaware statute authorized Delaware courts to take quasi in rem jurisdiction by sequestering property located within the state. Pursuant to this statute, P filed a motion to sequester Greyhound stock owned by several Ds, thereby invoking the courts jurisdiction in the present action. A Delaware statute set the stocks location in Delaware and the stock was seized. Ds challenged the courts jurisdiction on the ground that minimum contacts were lacking. The Delaware courts ruled that International Shoes constitutional test was inapplicable. Ds appeal. Issue: Does International Shoes constitutional test (fair play and substantial justice) apply to assertions of quasi in rem jurisdiction? Held: Yes. Judgment reversed. j Henceforth, all assertions of state court jurisdiction must meet International Shoes constitutional test. The test has expanded in personam jurisdiction, will have little effect on in rem jurisdiction, and will greatly affect the type of quasi in rem jurisdiction. j Ds only contact with Delaware is their stock in Greyhound which, because Greyhound is incorporated in Delaware, is deemed to be present in Delaware. The presence of such stock in Delaware is the sole basis for the courts exercise of jurisdiction, but the stock is totally unrelated to Ps underlying cause of action. There are not enough contacts between Ds, the forum state, and the underlying

21

litigation to satisfy the requirements of minimum contacts and reasonableness for the type of litigation before the court. j Property is only relevant if it fits in the International Shoe framework; the statute in Delaware making the stock property of Delaware is really a FICTION. The fact that the stock is not related to dispute that is being adjudicated is the major reason is doesnt fit under the minimum contacts framework. j If property is unrelated to the issue being adjudicated, could it amount to continuous and systematic contacts?  yes; if you are gaining benefit from the land (farm land, renting, doing business) than you are purposefully availing yourself to the benefits of the state. j Is is enough that hes a director in a Delaware corporation?  court says there must be minimum contacts, and being a director is enough for him to be adjudicated there, according to Brennan, who had a separate opinion, but only for cases that are relevant to the corporation he is the direct of

Comment: It is now settled that the constitutional test (fair play and substantial justice) and analysis are exactly the same for all types of personal jurisdiction cases.

World-Wide Volkswagon v. Woodson Facts: The Robinsons (Ps) purchased a new Audi from Seaway in New York. The following year, Ps were driving through Oklahoma, and Ps were severly burned from a car fire when they got into an accident. Ps filed products liability action against Seaway, Audi, Volkswagon, and World-Wide, alleging that the injuries to themselves and their children were caused by a defect in the car. Seaway, World-Wide, and Volkswagon entered special appearances, claiming that the courts exercise of in personam jurisdiction over them violated constitutional Due Process. The trial court ruled that it could exercise jurisdiction. Ds sought a writ of prohibition in the Ok. Sup. Court to restrain the judge (Woodson) from excercising personal jurisdiction over them. The OK supreme court affirmed the trial courts ruling even though there was no showing that any automobile sold by Seaway or World-Wide had ever entered OK. The court reasoned that an automobile is so mobile that Ds could have reasonably forseen its possible use in OK and that, given the value of the automobile, it was reasonable to infer that Ds derived substantial financial benefits form automobiles which form time to time are used in OK. SCOTUS granted certiori. Issue: Has the dealer or regional distributor of an automobile engaged in a purposeful act in OK by virtue of the fact that the automobile, sold in NY to NY residents, was involved in an accident while passing through OK.

22

j If you go by International Shoe contacts test you see that the contact is isolated and casual, but highly related probably would be jurisdiction j A court may exercise in personam jurisdiction over a D only if the D has purposefully availed itself of the forum states benefits and protections. Since D had no contacts with OK, it cannot be said that they purposefully sought to avail themselves of OKs benefits and protections. j The fact that the drivers drove the car to Oklahoma takes the causal factor away from the dealer. j Rejects the assertion that a consumers unilateral act of bringing the defendants product into the forum state was a sufficient constitutional basis for personal jurisdiction over the defendants. j Foreseeability of causing an injury in another State is not a sufficient benchmark for exercising personal jurisdiction; the forseeability that is critical to due process analysis is that the defendants conduct and connection with the forum state are such that he should reasonably anticipate being haled into court there. j When a corporation purposefully avails itself of the privilege of conducting activities within the forum state it has clear notice that is is subject to suit there, and can act to alleviate the risk fo burdensome litigation by procuring insurance, passing the expected costs on to customers, or, if the risks are too great, sever connections with the state. j Stream of water theory of jurisdiction D puts pollutants in the river in one state, and it carries down to another state would have jurisdiction in the 2nd state because it was foreseeable act. j Two functions of Due Process: 1) reasonableness inquiry: is it reasonableto adjudicate the case considering forum states interest in handling the dispute; the plaintiffs interest in obtaining effective relief, efficient resolution of conflict. 2) regardless to the issues of fairness, still need to have purposeful availment element. sovereignty function Dissent (Brennen): By focusing solely on the existence of contacts between the forum state and defendant, the Courts application of the minimum contacts rule is too narrow. The clear focus in Shoe teaches us that in addition to minimum contacts, the Court must look at the reasonableness of the assertion of jurisdiction Notes: -You are not domiciled until you establish residence in another state.

23

- For diversity jurisdiction to exist none of the defendants can be of the same state as the plaintiff. Asahi Metal v. Superior Court Facts: Man and wife severely injured in motorcycle accident in California, allegedly caused by the explosion of a defective tire. There he sued Cheng Shin, the Thai manufacturer of the tube. Shin impleaded Asahi, the Japanese manufacturer of the tubes assembly. The main claims were settled, leaving only Chen Sins (Ps) indemnity claim against Asahi (D). D moved to quash service. D made its valves in Japan and sold some to P in Taiwan, where D distributed the finished product all over the world, including California. The Cali. courts ultimately upheld personal jurisdiction as being within the states long arm statute and consistent with the due process clause. SC granted cert. Issue: Is Calis exercise of personal jurisdiction consistent with the constitutional due process? Held: NO. Judgment reversed j The long-arm Cali statute is the same as the Constitutional standard j Summary:  One the one hand, (4) justices suggested that minimum contacts would not exist over D because they were not purposefully and intentionally marketing their product. On the other hand, (4) Justices held that exercising jurisdiction would be unreasonable, considering the severe burdens on D of defending in a foreign legal system, the slight interest of P and California in the exercise of jurisdiction, and the international interest in not subjecting this alien corporation to an indemnification offshoot of a product liability action in America. They say that minimum contacts do exist, but jurisdiction is unreasonable. (1) justice says it depends on the volume, value, and hazardous quality of the product

What interests do you take into account when looking at reasonableness? - defendant - plaintiff - forum state Cheng Shin is a Taiwanese company. All that is left is 2 foreign parties. Have to be careful adjudicating rights of foreign company because it could be unfair, and there is public policy concerns.

24

OConner says that just injecting a product into the stream of commerce is not substantial enough to be subject to jurisdiction. Reasons are control-based; - the defendant should be able to benefit from the laws of the state - to be able to take steps to avoid litigation in a state

Comment: This is a rare case in which personal jurisdiciotn is held unconstitutional because, even though it passes the minimum contacts test, it fails the reasonableness test. Personal jurisdiction was unreasonable primarily because P settled the main lawsuit and has an obvious alternative forum for its secondary lawsuit against D (Taiwan or Japan) where both P and D have more substantial contacts than in California. Rule 4(k) allows the federal court to follow state laws unless there is an exception Burger King v. Rudewicz Facts: Burger King (P) is a Fla. corporation. Ds opened a BK in Michigan. P trains its franchisees and regulates their operations in detail, and regional offices supervise franchisees in their areas. Ds franchise contract was negotiated mainly with the district office, but also with Miami headquarters. (FINISH FACTS) Issue: Did the district courts exercise of personal jurisdiction over Ds offend due process? Held: No, judgment reversed Two stages for minimum contact 1) Minimum contacts: The Due Process Clause protects and individuals liberty interest in not being subject to binding judgments of a forum with which he has established no meaningful contacts, ties, or relations. Thus, a defendant must purposefully establish minimum contacts within the forum state. 2) Reasonableness:Notion of fair play and substantial justice take into account o the burden on the defendant o the forum states interest in adjudicating the dispute o the interstate judicial systems interest in obtaining the most efficient result o share interest of the several states in furthering fundamental substantive social policies. *** Court says that if the reasonableness factors are very strong, than it can reduce the importance of minimum contacts*** - Sounds like Brennans dissent in WWVolskwagon.

25

What effect does strong contacts have on the reasonableness factor if the D wants to show there is no jurisdiction? - have to come up with very good evidence of unreasonableness to avoid jurisdiction - weak minimum contacts, and strong reasonableness would be sufficient to assert jurisdiction - strong reasonabless and weak minimum contacts would also be sufficient to assert jurisdiction. - Both ideas are separate, but inter-related j Ds have had no physical ties to Fla. save for a brief managerial training course in Miami. Yet this franchise dispute grew directly out of a contract that had a substantial and continuing connection with the state. Ds reached out to negotiate with a Fla. corporation and agreed by long-term contract to be regulated form Fla., to make payments to Fla. and to have disputes goverened by the laws of Fla.

Dissent: Says there is a significant element of unfairness in requiring a franchisee to defend a case of this kind in the forum chosen by the franchisor. (unlike McGee where the smaller party chose to bring suit)

GENERAL JURISDICTION For technology purposes Paige thinks that there shouldnt be a technology specific test; could apply old principles to assert jurisdiction. Sliding Scale used for internet cases One end of the spectrum are situations where a defendant clearly does business over the internet. If a defendant enters into contracts with residents of a foreign jurisdiction that invoke the knowing and repeated transmission of computer files over the internet, then personal jurisdiction is proper. At the opposite end are situations where a defendant has simply posted information on the web site which is accessible to users in foreign jurisdictions. A website that just makes information available to those who are interested in it is not grounds for the exercise of personal jurisdiction. The middle ground is occupied by interactive websites where a user can exchange information with the host computer.

26

Pavlovich v. Superior Court Facts: P (located in California) is an organization that created a program to block copying DVDs. D is from Texas, and he creates a website that gives the code to decrypt the DVDs so they can be copied. Issue: Did the California Sup. Court properly exercise personal jurisdiction over D based solely on a passive website? Held: No, judgment reversed. Application: Court says you can look at it two ways: 1. sliding scale test 2. effects test Sliding Scale test: Says that it is just an informational website. No evidence that the site had been visited, that anyone got the information. Totally passes the sliding scale test. Effects Test: Says D should have known that his actions would have harmful consequences to the P in the forum state; two prongs (i) D expressly aimed his tortious conduct at the forum state and (ii) be aware that his intentional conduct would cause harm in the forum state. In this case, D did not expressly aim his tortious conduct at California. There is no evidence that any Cali resident ever visited the website, much less downloaded the program. j D did not expressly aim his tortious conduct at California. Ds sole contact with Cali was posting a program containing Ps proprietary information on the website which any person could have access to. j Three prongs are required to exercise specific jurisdiction over a nonresident defendant: (i) D has purposefully availed himself of forum benefits- the purposeful availment test (ii) the controversy is related to or arises out of Ds contacts with the forum; and (iii) the assertion of personal jurisdiction would comport with fair play and substantial justice. This case turns on purposeful availment.

Comments: The majority opinion is consitent with the view in most jurisdictions that the mere presence of a website is insufficient to establish personal jurisdiction, in states where the web site can be viewed. Either exchange of information or business is needed to establish express aiming and justify the exercise of specific jurisdiction based on the website alone.

General Jurisdiction

27

There is no need to go to the reasonableness inquiry if you satisfy the requirements for general jurisdiction. General Jurisdiction: with unrelated contacts, they have to meet higher standard of frequency; need continuous and systematic contacts.

Coastal Video Comm. v. Staywell Corp. Facts: P (located in Virginia) has book on Defending your Safety Zone, and sues D (a Deleware corp. that does business in Cali) for a declaratory judgment in Virginia that its use of the phrase, Your Safety Zone does not infringe on copyrighted material owned by Ds. Ds motioned for lack of personal jurisdiction pursuant to Fed. Rule 12(b)(2). In response P filed a motion seeking to engage in additional discovery regarding the nature and extent of Ds contacts with the forum state, Virginia. Looks to different contacts that D has in Virginia. The connections are that D has sold products to hospitals in Virginia, sent mailings to Virginia residents, and had an interactive website that advertises and sold products in Virginia, BUT there is no evidence that particular book has been sold in the state. Issue: Should Ps motion for additional discovery on the issue of general jurisdiction over D be granted prior to considering Ds motion to dismiss for lack of personal jurisdiction? Held: Yes. j Rule 12 (b)(2) permits dismissal of an action lacking the requisite personal jurisdiction. j Not specific jurisdiction b/c the book in question was never shown to be sold in the state. j A court may exercise personal jurisdiction over a defendant if it has general jurisdiction, in which the requisite minimum contacts between the defendant and the forum state are fairly extensive. Conduct of single or isolated items of activities in a state in the corporation's behalf are not enough to subject it to general jurisdiction. Even continuous activity of some sorts by a corporation within a state is not enough to support general jurisdiction over the corporation. Only when the continuous corporate operation within a state is thought so substantial and of such a nature as to justify suit against it on causes of action arising from dealings entirely distinct from those activities may a court assert general jurisdiction over a corporate defendant. Broad constructions of general jurisdiction should be generally disfavored. j General jurisdiction clearly exists only when the cause of action does not arise from defendant's contacts with the forum state. General jurisdiction is defined as the exercise of personal jurisdiction over a defendant in a suit not arising out of or related to the defendant's contacts with the forum.

28

j When conducting a general jurisdiction analysis, it is not enough to find that an interactive website has the potential to reach a significant percentage of the forum state's population. Instead, for the contact to be continuous and systematic, there must be proof that the website is actually reaching a portion of the state's population. The mere existence of an interactive website, without proof that there was continuous and systematic contact between the forum state and website, is not sufficient for a court to exercise general jurisdiction.

PERSONAL JURISDICTION REVIEW/ APPROACH ANALYSIS(IMPORTANT FOR EXAM!!) Ultimate question is where it is appropriate to file a case Depends on the connection between the defendant and the state; here is a step by step analysis. 1. Look at contacts between defendant and state; look at physical presence in state (can anticipate the defendant being in state at particular time he can be subject to personal jurisdiction) business, property, etc. Then ask whether contacts are sufficient. j he likes to see a capsule summary of what test you are trying to apply; e.g. for Intl Shoe explain the chart on pg. 17 of notes. i. what is the first issue you would look at, in asserting jurisdiction over out of state defendants? 1. state long-arm statute 2. would it be constitutional for state to do so (due process)? a. with minimum contacts what is the question? Whether there is purposeful availment.This is what court is talking about when they talk about forseeability of being haled into court. THESE ARE ALL MINIMUM CONTACTS TESTS! j situations you might see this is when there is a contractual relationship across state lines; Hanson v. Denkla, Asahi, Burger King j Asahi (stream of commerce) have to forsee being haled into court; OConner says that if all you have is pure stream of commerce that is not enough for jurisdiction, in addition to this you need in addition to advertising, marketing, something designed for the state. Taking purposeful to its most literal extent; four votes said the opposite that pure stream of

29

commerce is enough for personal jurisdiction; Stevens, the swing vote, looks at the hazardousness of the sales. j With the internetcases, the court uses the effects test; look at the nature of the website. b. Reasonable inquiry; it is possible to have the reasonableness inquiry trump the minimum contacts test. j Reasonableness:Notion of fair play and substantial justice take into account - the burden on the defendant - the forum states interest in adjudicating the dispute - the interstate judicial systems interest in obtaining the most efficient result - share interest of the several states in furthering fundamental substantive social policies. *** Court says that if the reasonableness factors are very strong, than it can reduce the importance of minimum contacts***

Burnham v. Superior Court Facts: (P) brought suit for divorce in Cali state court against her husband (D). D, a resident of New Jersey, was served with process at Ps home in California while visiting on business to see his children. D returned to NJ, but later made a special appearance in a Cali trial court to file a motion to quash service. The motion was denied, and the Cali appellate court affirmed. SCOTUS granted cert. Issue: Is it consistent with constitutional due process for California to exercise personal jurisdiction over a nonresident who was personally served with process while temporarily in Cali. for a claim tat is unrelated to his in-state activities (i.e. is transient jurisdiction constitutional?) Held: Yes [if you apply Intl Shoe to the pure minimal situation of transient jurisdiction; if you were just driving through the state, under minimum contact standards you would argue that it was both unrelated and isolated and casual.]

30

y Summary: While the decision upholding transient jurisdiction was unanimous, the reasoning was not. Four Justices believe that transient jurisdiction comports with traditional notions of fair play and sub. justice simply because it has been consistently upheld since the 19th century and is supported in consensus by state judges today. Four other Justices believe transient jurisdiction passes the fairness test, b/c it satisfies both the minimum contacts and reasonableness sub-tests. y (Scalia, Rehnquist, CJ, White, Kennedy). Jurisdiciton based on physical presence alone constitutes due process because it is one of the continuing traditions in our legal system that defines the due process standard of traditional notions of fair play and substantial justice y (Brennan, Marshall, Blackmun, OConner). Minimum contacts are established b/c a transient D actually avails himself of significant benefits provided by the state. Modern transportation and communications have made it much less burdensome for a party sued to defend himself. And any burdens that do arise can be ameliorated by a variety of procedural devices. Scalias response to Brennans opinion: Even if you could argue that this guy had minimum contacts, you dont want to adopt a rule that says you need to go through this analysis every time you use tag jurisdiction, because to engage in the inquiry that Brennan goes into causes a great deal of uncertainty. (i.e. say the children were sick, and the husband came to visit them) Scalia argues for CERTAINTY on the issue. j cant get somewhere to a state by fraud, but if them come voluntarily they can be served with process.

CONSENT AS A BASIS FOR JURISDICTION Implied consent is really no longer part of the analysis, consent is now imposed by the law based on the minimum contacts; BUT it doesnt mean that express consent is totally ruled out. Intro: Parties may voluntarily, knowingly, and intelligently consent to personal jurisdiction in advance of litigation even though the exercise of jurisdiction would not otherwise comport with minimum contacts. However, consent jurisdiction is still subject to judicial scrutiny for fundamental fairness. Carnival Cruise Lines v. Shute Facts: The Shutes (P) purchased a ticket for a seven-day cruise on Carnival Cruise Lines (D). P booked through a Washington travel agent. Each ticket said on it that all disputes arising under it would be settled in Florida state court (the forum-selection clause).

31

While on the ship, P slipped on a deck mat and was injured. P sued in Washington Federal court claiming negligence on Ds part. D moved for summary judgment based on the forum-selection clause in the ticket saying that all cases must be adjudicated in FL. Issue: Did the court of appeals err in refusing to enforce the forum-selection clause? Does a tort that occurs outside of the state, arise out of the transaction of business that occurs in the state when looking at the long-arm statute? [states differ in their long arm statute interpretation concerning a tort that arises out of the state from a business transaction within the state usually dont favor this] Held: Yes, judgment reversed j The inclusion of a forum-selection clause contained in a passage contract is reasonable because it limits the forums in which D may potentially be sued; it dispels any uncertainty as to where suits arising out of the contract can be brought; and in this case, it reduces the fares. j Although otherwise reasonable, forum-selection clauses contained in passage contracts are subject to judicial scrutiny for fundamental fairness. In this case, the forum-selection calsue passes the fairness test b/c there is no indication that D selected FL as a means of discouraging legitimate claims. D selected Florida simply because its principal place of business in FL. Ps conceded that they were given notice in the forum-selection clause. - If the forum selection clause said that all cases would be adjudicated in Singapor this would be more likely to be interpreted to discourage law suits.

THE CONSTITUTIONAL REQUIREMENT OF NOTICE Going back to Pennoyer court was concerned with both power, consent, and notice. Whenever you serve process within the boundaries in the state under Pennoyer you are asserting both power and notice. Once you move to the Intl Shoe framework where the power requirement is settled by the minimum contacts tests, NOTICE becomes a separate issue. Service is no longer use for basis of power, it is now used for NOTICE.

Mullane changed Pennoyer regime of notice almost as drastically as Intl Shoe changed the Pennoyer regime of personal jurisdiction. First, the requirement of notice is no longer dependent upon the old classifications, and second, the present constitutional role concerning the adequacy of notice is that the notice must be reasonably calculated,

32

under all circumstances, to apprise interested parties of the pendancy of an action and afford them the opportunity to present their objections.

Mullane suit brought by the bank who is the trustee of all the accounts- the trustee says that I have been managing the trust, and distributing interest to some of the beneficiaries, and this is everything that I have done,the bank is asking the court to look back upon what they have done and either approves/disapproves. If court provides a settlement of trust account, it gives the trustee the ability to have certainty of not being subject to lawsuit for the investments that they have made. Once the court has said that the managemement of the fund has been prudent, it cuts off all the rights of the beneficiaries to sue for breach of fiduciary duty. Raises the issue, Does the court in these circumstances, have jurisdiction over the beneficiaries to adjudicate their rights? Mullane v. Central Hanover Bank Facts: Pursuant to NY Banking Law, Central Bank petitioned the court for settlement of its account as trustee of a common trust fund that had 113 participating trusts. IN strict compliance with the banking law, the bank notified the beneficiaries of the trust in a local newspaper. State required notification by publication (standard notice of providing in rem notice) Mullane was appointed special guardian and attorney for all known and unworn present and future income beneficiaries of the common trust fund. Many of the beneficiaries were non-residents. Mullane objected to the statutory requirements as violative of constitutional due process. The state courts overruled Mullanes objection and a final decree accepting the accounts was entered. Mullane appealed. Issue: Does the statutory requirement of publication violate due process when the whereabouts of some fo the beneficiaries so served are known? Held: Yes at to the known beneficiaries, no as to the unknown beneficiaries. j Notice must be reasonably calculated, under all the circumstances, to apprise interested parties of the pendency of the action and afford them the opportunity to present their objections. j Court says most is required is a reasonable method of notice, and it is required when there is a known trustee; personal service is the best. But sometimes, a reasonable method is not available, and in that case, bring a form of notice chosen is not substantially less likely to bring home notice than other of the feasible and customary substitutes j Publication falls short of this goal with respect to the known beneficiaries, i.e. beneficiaries whose names and addresses are known and most of whom are residents of the forum state. Notification by mail is the best means possible for such beneficiaries. Dont have to be personally served.

33

j Publication is not only the best but also the only menas possible of notifying the unknown beneficiaries, i.e. beneficiaries whose interest in the trust are remote or whose names or whereabouts cannot be ascertained through reasonable efforts. j If you have a pure in rem action, the publication is enough, because it would be brought to your notice if your property was seized. In later cases the court has made clear, that regardless of the proceedings, if you know where the person is you have to send them notice by mail, or another reasonable method. j Really Mullane broke the minimum requirement of personal service, broke the notion that personal service is always required. Usually other ways are available. Class notes: Court says that they have power b/c even though they are relying on property they are really adjudicating individual rights. Element of necessity, if you are going to have trust funds, you have to have a way to adjudicate these procedures. Regardless of you perceive the proceedings are in rem or in personam, they have power to hear the case, as long as there is NOTICE.

Rule 4 is the service of process rule:Need to have the statutory authorization, and then you have to look if the service of process meets the constitutional standard. Need to comply with both, or it might be dismissed. Its similar to the long-arm/constitutional relationship for jurisdiction. READ RULE 4 CAREFULLY!!!! Rule 4: if you are being served by a court in Florida, and the person being served is in NY, you can use either Fla. or NY methods to give service. 4(K)(b): provides you can reach 100 miles from the actual court (across state lines even if not authorized) to serve process 4(K)(c): bypasses all questions of personal jurisdiction, just subject to jurisdiction where case is filed. 4(K)(d): Clayton Act; allows worldwide service of process 4(K)(2): When would it be constitutional to assert jurisdiction when a party has no minimum contacts with a state? - the federal court in the U.S. can assert personal jusrisdiction over a foreign corporation if that corporation has any contacts with the United States, doesnt need to be with a specific state. 4(L): requires proof of service

34

4(M): Service must be accomplished within 120 days, or the case will be dismissed without prejudice (it can be heard again)

VENUE
Venue is purely statutory provision that tells you where the case will be filed. Federal venue statute is 28 U.S.C. 1391 a. The venue concept. Venue simply means the place of trial. Venue rules attempt to allocate cases within a judicial system, federal or state, in a manner that is convenient for the parties, witnesses, and the court. Typically, questions of personal and subject matter jurisdiction are decided in advance of the venue. The venue rules are statutory b. Federal Venue rules: The general venue rules for federal courts are set out in 28 U.S.C. section 1391. There are 4 types of general venue rules; 1) diversity actions; 2) federal question actions; 3) corporations; 4) aliens. Diversity Venue Section 1391(a): Venue is proper only in the jurisdiction where; (1) any defendant resides, provided that all defendants reside in the same state (2) any substantial part of the events, omissions, or property concerning the controversy is situated; or (3) the Ds are subject to personal jurisdiction at the time the lawsuit is commenced, if there is no district in which the action may otherwise be brought. Federal Question Venue 1391(b). Venue is proper only in the jursidciton where; (1) any defendant resides, provided that all defendants reside in the same state (2) any substantial part of the events, omissions, or property concerning the controversy is situated; or (3) a judicial district in which any defendant may be found, if there is no district in which the action may otherwise be brought. Corporations 1391 (c) (1) For purposes of applying sections (a)&(b), a corporation is deemed to reside in any judidical district where it is subject to personal jurisdiction

35

(2) If there is more than one judicial district in the state, then the corporation is deemed to reside in any judicial district in that state within which its contacts would be sufficient to subject it ot personal jurisdiction if that judicial district were treated as an independent state. 1. If no such judicial district exists, then the corp. shall be deemed to reside in the judicial district within which it has the most significant contacts. Explanation of 1391 C(2) Says that you have more than one district with a corporation, you bring the suit under the jurisdiction where the corporation has the contacts. If the corp. has contacts with the entire state and they are insufficient for one in particular district, go to the district that is has the most significant contacts with.

Examples from pg. 165 (answers) 1. Venue would certainly be proper in Southern D. of NY because thats where the defendant resides 1391(a) . ALSO since design occurred in New Mexico it was a substantial part of the events that gave rise to the claim, so you can bring it in New Mexico. Also can bring in Illinois for the same reason. Say there is a second resident that is a resident of Brooklyn, where would venue be proper then? In either district, because you can have jurisdiction where either defendant resides. 2. Apply (a)(3) probably NY because they are both doing business in NY

Dee-K Enterprises v. Hevafil Facts: Dee-K and Ashboro (Ps) Virginia and NC corps, sued several Asian and American corps (Ds) in a Virginia federal district court, alleging an international conspiracy to restrain trade, and fix prices of rubber threat in the U.S. Ps were buyers and Ds were manufacturers and distributors of rubber thread. Ds challenge jurisdiction and venue. Issue: (i) Does the court have personal jurisdiction over the Indonesian defendant who manufactured and consummates its sales of rubber thread in Indonesia? (ii) Is the Eastern Dist. of Virginia the proper venue? Held: (i) Yes (ii) Yes, as to the Asian Ds, and maybe as to the American Ds.

36

j The Clayton Act-Sec 12 (Rule 4K(2)) says if all defendants have minimum contacts with United States as a whole they can have personal jurisdiction, and such statutory reach is constitutional because Ds have in various ways purposefully availed themselves of the U.S. markets j Section 1391(d) provides that aliens (including foreign corporation) may be sued in any district, and it overrides the Clayton Acts special venue provision which fixes venue in any district where a defendant is found or where it transacts business. None of the Asian Ds can be found or transact business in the East. Dist. of Virginia. j But as to the American Ds venue is fixed by 1391(b)(3) since neither b1 (b/c not all Ds were residents of state) nor b2 (because the events did not occur inside the state) applied. Under b3, venue is proper where a D can be found. The parties must make additional factual showings to determine whether the American Ds conducted sufficient activities in the E. Dist. of Virginia to be found there. DECLINING JURISDICTION OR VENUE Transfer Venue:1404Allows a jurisdiction that has personal/and subject matter jurisdiction to transfer a case as long as the other district has venue. j Occurs when venue is proper but inconvenient. j Have to show that venue was proper both in case where case was originally filed and in the new district where it will be transferred to. j After the transfer, the law of the original forum will be used. j the transfer must be convenient for the partiers and witnesses, and in the interest of justice.

1406- says that even if case is filed in wrong district, it can be transferred and the transferees court law applies.

Forum Non conveniens: Forum non conveniens is a product of the common law doctrine which says that it is our inherent authority whether we want to hear a case or not. It differs from 1404 in these ways; j state courts cannot use 1404, while all courts can use forum non conveniens; wouldnt use a transfer for forum non conveniens in a federal court, but you can use it in a federal court with respect to international issues. j 1404 is a transfer of the same case, the case is not dismissed; forum non conveniens requires dismissal of case, and the case needs to be refiled in foreign jurisdiction. j 1404 allows transfer by motion from P or D. Forum non conveniens can only be motioned for by D. j More deference to the Plaintiffs choice of venue in forum non conveniens, rather than 1404 Gilbertis the leading case on this.

37

j The private factors include; relative ease of access to sources of proof availability of compulsory process for attendance of unwilling, and the cost of obtaining attendance for the willing witnesses. Only if the private factors balance out do you go to the public interest factors. j The public factors include, administrative difficulties from court congestion, the local interest of having local questions decided at home, the avoidance in conflict of different law of different states/countries, unfairness of burdening citizens in an unrelated forum with jury duty.

Forum Non Conveniens in Federal Court Piper Aircraft Co. v. Reyno Facts: An aircraft manufactured by Piper in Pennsylvania and Hartzell in Ohio (Ds) crashed in Scotland, killing 5. The decedents were Scottish residents. At the time of the crash the plane was registered, owned, and maintained in England by an English company and operated by a Scottish air taxi service. The wreckage of the plane was in England. Reyno (P), filed separate wrongful death actions against Ds in California Court as the court-appointed administratrix of the decedents estates. Decedents survivors field a separate action in England against the pilot, the owner, and operator of the aircraft. P admits that her lawsuit was filed for the sole purpose of taking advantage of the strict liability law, which is recognized under U.S., but no Scottish law. After case was removed to a federal district court in Cali, and then transferred using 1404 to PA, Ds moved to dismiss on the grounds of forum non conveniens. The district court granted the motion but was reversed by the court of appeals on the ground that a forum non conveniens motion could be defeated by a mere showing that the substantive law that would be applied in the convenient forum is less favorable to the plaintiff than the present forum. Ds appeal. Issues: (i) (ii)

Should the possibility of an unfavorable change in law, by itself, bar dismissal on forum non conveniens grounds? If not, did the district court otherwise abuse its discretion in granting dismissal on forum non conveniens grounds?

Held: (i) No. (ii) No. Judgment reversed. j The possibility of an unfavorable change in the substantive law ordinarily should not be given conclusive or even substantial weight in the forum non conveniens decision; otherwise, the doctrine would lose of of its flexibility and, in fact, would become virtually useless. Court is saying you cant give decisive weight to this factor because the plaintiff always chooses a district with more favorable law. j On the other hand, if the remedy provide by the alternative forum is so clearly inadequate that is it no remedy at all, the unfavorable change in law may ne given

38

substantial weight; the district court may conclude that dismissal would not be in the interest of justice. Here, the remedies provided by the Scottish law dont fall within this category. j The forum non conveniens determination is committed to the sound discretion of the trial court and may be reversed only where there is a clear abuse of discretion. j In granting Ds motion, the district court relied on the balancing tests set fort in prior cases. In these cases the court said that there is ordinarily a strong presumption in favor of plaintiffs choice of forum, which may be disturbed only when the private interests (affecting the convenience of the litigants) and the public interest (affecting the convenience of the forum) clearly point toward trial in the alternative forum. The presumption applies with less force when the plaintiff is foreign, because the central purpose of any forum non conveniens inquiry is to ensure that the trial is convenient when a plaintiff selects a foreign forum it is not reasonable to assume that this choice is convenient to such a plaintiff. ***IN THE FEDERAL SYSTEM, ANY TIME THERE IS SOME REMEDY AVAILABLE THAN A CASE CAN BE DISMISSED FOR FORUM NON CONVENIENS*** This is international case thats why there is forum non conveniens.

FEDERAL SUBJECT MATTER JURISDICTION


CAN SHOW EITHER FEDERAL QUESTION OR DIVERSITY JURISDICTION TO GET INTO STATE COURT!!!!!

Federal Question Jurisdiction: 28 U.S.C. 1331The district courts shall have original jurisdiction of all civil actions arising under the Constitution, laws, or treaties of the United States. Statutory Basis and Interpretation: Title 28 U.S.C. section 1331 gives the federal district courts orginal jurisdiction of all civil actions arising under the Constitution, laws, or treaties of the United States. The arising under phrase has resisted all attempts to frame a single, precise test or definition for determining which cases fall within or outside federal question jurisdiction. In the following case, the Court considers several tests and finally relies on the well-pleaded complaint rule. This rule states that whether a case arises under a federal law (statuory or common law) must be determined from what necessarily appears in the plaintiffs statement of his claim in the bill or declaration, unaided by anything alleged in anticipation of avoidance or defense which it is thought the defendant may impose. An important implication of this rule, has to do with removal jurisdiction: a defendant may remove a case from state court to federal court only if plaintiffs complaint established that the case arises under federal law.

Well Pleaded Complaint Rulegoverns the case below. This rule states that whether a case arises under a federal law (statuory or common law) must be determined from what necessarily appears in the plaintiffs statement of his claim in the bill or declaration,

39

unaided by anything alleged in anticipation of avoidance or defense which it is thought the defendant may impose. The rule further says that a suit arises under the Constitution and laws of the United States only when the complaint shows that it is based upon those laws or that Constitution. It is not enough that Ps allege some anticipated defense to the lawsuit and assert that the defense is invalidated by some provision in the Constitution.

Want the Federal Courts hearing the most essential Federal Cases; need a screening process for cases where the federal issue is not in the forefront. Cases below deal with this concept

Louisville & Nashville Railroad v. Mottley Facts: The Mottleys (Ps) sued Louisville & Nashville Railroad (D) in federal court seeking specific performance of a settlement agreement arising out of a railway accident in which Ps were injured. Under the settlement, D granted Ps lifetime passes for free transportation, which Ps used for a number of years. Ds honored the passes then refused to do so after federal legislation forbidding railroads from giving free transportation came out. Ps argued alternatively that the Act of Congress did not apply to them or that if it did, it violated the 5th amendment prohibiting against the govt taking property w/o due process. The trial court ruled for Ps, Ds appeal. Issue: Does this lawsuit fall within federal subject matter jurisdiction? Held: No, judgment reversed j There was no diversity of citizenship between Ps and Ds but it was alleged that the case was a suit.. arising under the Constitution and laws of the United States. So if jurisdiction exists, it can only be based on section 1331, federal question jurisdiction. j A suit arises under the Constitution and laws of the United States only when the complaint shows that it is based upon those laws or that Constitution. It is not enough that Ps allege some anticipated defense to the lawsuit and assert that the defense is invalidated by some provision in the Constitution. j In this case, Ps do not allege a claim, arising under federal law. They merely allege that Ds defense to their claim for specific performance violates federal law i.e. that Ds defense arises under federal law. j 12(h)(3) : defense of lack of subject matter jurisdiction can never be waived. Notes: Even if the issue of subject matter jurisdiction doesnt come up in the federal courts, then it is just assumed that the court has jurisdiction Up through the appeals process the issue subject matter jurisdiction can be raised at any time; but if the case is adjudicated then it is final.

40

Why is it necessary for the Federal Court to have a filter? - cases where federal rights are viewed unfavorably by local courts, so you want to have a federal forum that is sympathetic to federal rights - federal court judges appointed by president for life, insulated from local political pressure - Supreme Court cant review every single case regarding a federal issue; have to have some way to provide a separate forum that is sympathetic to state rights. A demurrer is the motion to dismiss a claim saying that the Plaintiffs are not stating a legally sufficient claim. When a defendant challenges federal question jurisdiction, one of three questions commonly arises: 1. is there a federal question at all. 2. if there is a federal issue does is give rise to Ps claim? 3. is it sufficiently important to federalize the case? For diversity cases use Rule 12(b)(1) only as a means of dismissal for lack of subject matter jurisdiction. A party arguing that a complaint does not state a claim arising under federal law could move for dismissal invoking either Rule 12(b)1 or Rule 12(b)6. Addressing this situation, the Court has said that if theres any arguable basis for a federal claim, the district court should examine the federal question not as a matter of jurisdiction but on a Rule 12(b)(6) motion to dismiss the substantive claim. Example Question top of pg 182. 1) the case arises under the statute, and although the only issues to resolve are issues of fact, the case is arising under Federal Law applying the well-pleaded complaint rule. (problem here is that it falls under federal jurisdiction when there is no important federal issue of law) 2) claim arises under state tort law of libel, doesnt arise under federal law; the first amendment allegations are in anticipation of a defense that the defendants were going to use. In general if there is a misinterpretation of federal law in a suit, it is still a federal issue arising under the law. DIVERSITY JURISDICTION 28 U.S.C. 1332 is the main rule for diversity of citizenship. *** KNOW THIS STATUTE****

41

28 U.S.C. 1332 (a) The district courts shall have original jurisdiction of all civil actions where the matter in controversy exceeds the sum or value of $75,000, exclusive of interest and costs, and is between (1) citizens of different States; (2) citizens of a State and citizens or subjects of a foreign state; (3) citizens of different States and in which citizens or subjects of a foreign state are additional parties; and (4) a foreign state, defined in section 1603(a) of this title, as plaintiff and citizens of a State or of different States. Statutory Basis: Diversity jurisdiction brings nonfederal (state) claims into federal court. Diversity jurisdiction is permitted by Article III of the Constituion and has been established by 28 U.S.C. 1332. Unlike federal question jurisdiction, diversity has an amount in controversy requirement: the amount must exceed $75,000 Diversity for Corporations: 28 U.S.C. section 1332(c) defines citizenship of a corporation as the state of incorporation and the state in which its principal place of business is located. Many corporations are citizens of Delawares permissive laws. j The standard formulation says that state citizenship depends on present domicile and intent to remain indefinitely. That test is easy to apply for most people who have a single clear affiliation. j -1332 requires that each plaintiff must be diverse from each defendant. j -Two citizens from foreign countries cannot have diversity jurisdiction. j -Partnerships are considered a group of individuals, so if anyone in the partnership is from the same state as defendant, the case will be thrown out.

j Suits for alimony, child custody, are outside the scope of diversity jurisdiction, even if spouses are citizens of different states.

Redner v. Sanders Facts: Redner (P), a U.S. citizen residing in France, sued Sanders, and others (Ds), all citizens of New York, in the district court for the S. District of NY. P asserts federal jurisdiction based on diversity citizenship, based on 1332 (a)(2). The complaint avers that diversity jurisdiction exists b/c P is a resident of a foreign state, while Ds are residents

42

of the State of NY. Ds move under Fed. Rule 12(b)(1) to dismiss for lack of jurisdiction. Issue: Are Ps and Ds diverse within the meaning of the diversity statute? Held: motion granted. j 28 U.S.C. 1332 provides in part (a) The district courts shall have original jurisdiction of all civil actions where the matter in controversy exceeds the sum or value of $75,000, exclusive of interests and costs., and is between (1) citizens of different states; (2) citizens of a State and citizens or subjects of a foreign state. j P seeks to invoke subsection (a)(2) as a basis for jurisdiction. However, Ps complaint speaks of residence whereas the statute defines diversity in terms of citizenship. The two are not synonymous. For jurisdiction to exist under (a)(2), P would need to be a citizen of a foreign state, not merely a resident. The complaint itself alleges that P is a citizen of the U.S. Thus, the case does not involve an action between citizens of a state within the United States and a citizen of a foreign state. No jurisdiction exists under 1332(a)(2). j In response to the 12(b)(1) motion, P shifts ground and asserts that he is domiciled in California. This is an apparent attempt to lay a basis for jurisdiction under 1332(a)(1) jurisdiction based on citizens of different states. However, Ps factual submission is not sufficient to demonstrate a California domicile. Notes: If you are not domiciled in the United States, and you are not a citizen of a foreign country, you have taken yourself out of diversity jurisdiction. Diversity for Aliens. Federal courts do not have diversity jurisdiction over a lawsuit between foreign nations (e.g., a French national vs. an English national) if neither has U.S. immigrant status. Such suits must be brought in state court. If one of the parites has achieved permanent resident status (although not U.S. citizenship) 28 U.S.C. 1332(a) gives her the status of a ciizen of the state which she is domiciled, as the next case illustrates.

Saadeh v. Farouki Facts: Saadeh (P) sued Farouki (D) for breach of contract in federal district court, relying on diversity jurisdiction. P was a Greek citizen. D was a Jordanian citizen who become a permanent resident alien in the United States residing in Maryland. The district court ruled it had diversity jurisdiction. D appeals. P(Greece) D (Jordanian) domiciliary of Maryland / D corp. D.C.

43

Issue: Under the 1988 alienage amendment to 28 U.S.C. 1332(a) (which states an alien admitted to the U.S. for permanent residence shall be deemed a citizen of the state in which such alien is domiciled), does a United Statesdistrict court have diversity subject matter jurisdiction in a suit between a nonresident alien and a resident alien? Rule:There needs to be complete diversity between each plaintiff and each defendant!!! There has to be diversity between each plaintiff and each defendant. j In an effort to limit diversity they treat citizens who are domiciliaries of a state as citizens of the state to DEFEAT diversity. BUT court says that when it comes to creating diversity they dont treat an alien resident as a citizen of the state. j This is an odd case. First the literal reading of the statute is not only at odds with congressional intent, but would also lead to an absurd result. The last sentence of section 1332(a) now means that permanent resident aliens are deemed to be citizens of states for purposes of defeating diversity jurisdiction (the amendments intent was to deny diversity in lawsuits between citizens and resident aliens of the same state), but are not deemed to be citizens of states if that would expand diversity jurisdiction (e.g. lawsuits between aliens, residents or not, as in this case. j The fact that D gained citizenship during the suit does not matter, because its the citizenship at the time the suit commenced. j Citizenship would not matter in case of Federal Question jurisdiction. CORPORATIONS Corporations have dual citizenship; both the place where they are incorporated and where it has its chief place of business. j If it is incorporated in more than one state, it has citizenship in ALL those states. j only can have citizenship for the chief place of business part in ONE state. j Corporation can be a resident of any state which it has personal jurisdiction in, but it is only a citizen of one or two states.

Corporations- Two tests for CHIEF PLACE OF BUSINESS :

nerve center test look at where the decision-making on a day to day basis is made corporate activities- where the bulk of the business is done.

44

total activities test production, sales, decision making, and see where the largest amount of the total activities take place.

If you have a P (FL) v. D(FL) D(GA) : this situation is not diversity jurisdiciotn P (FL) v. D(GA) D(GA) : fact that you have two parties on same side of litigation does not matter. j Only look for across diversity. Exceptions to the rule for complete diversity. The Federal Interpleter Act 1335 allows for diversity jurisdiction as long as there is one P/D from different states even if there are Ps and Ds from same state.

AMOUNT IN CONTROVERSY There is no amount in controversy for Federal Question cases. Congress has continually increased amount in controvery over the years, now it is $75,000. Can you sue for damages in excess of $75,000 and get diversity jurisdiction? y It has to appear to be a legal certainty that the amount in controversy will be less than the jurisdictional amount. Has to be a legal certainty that you cant recover that amount. It is a lenient standard. y If you are one plaintiff v. one defendant you can aggregate all your claims against the defendant regardless of how related they are. But if you are two separate claims against the same person you cant aggregate for purposes of this rule, unless there is a common interest.

Example: P(FL) ($75,000) & P(FL) ($5,000) v. D(GA) y Supreme Court ruled that this situation is permissible so long as you have complete diversity. Then if one of the Ps meets the jurisdictional amount requirement it brings in all the other Ps to diversity jurisdiction as long as it is the same case.

45

SUPPLEMENTAL JURISDICTION Supplemental jurisdiction codifies in 28 U.S.C 1367 two common law concepts. pendant and ancillary jurisdiction. y Pendentjurisdiction allows a plaintiff to join a state claim with a federal question claim in her complaint. If you have both a federal and state claim, you can bring in the state claim along with the federal claim in federal court if they are rising from the same transaction or occurrence. Both claims are asserted against the same defendant. The district court has pendant subject matter jurisdiction over the nonfederal claim. o even if (P) has a federal claim against D1 and a state claim against D2 it can still be heard in federal court under supplemental jurisdiction. the court has the option to dismiss the state claim.

Rule from United Mine Workers v. Gibbs y Pendant jurisdiction (now supplemental jurisdiction under 1367) may be exercised only when the relationship between the federal and nonfederal claims is such that it can be said that the entire lawsuit before the court comprises but on constitutional case. This relationship exists when both claims, derive from a common nucleus of operative fact. y If the federal claims are trivial and there is overriding state claims, than the federal court would probably not hear the case y Federal question can also be dismissed on a 12(b)(6) motion. 1367(a) reflects the power notion of Gibbs. Allows the assertion of supplemental jurisdiction, allows a state claim to be heard with a federal claim if they are part of the same constitutional case. o 1367(a) goes beyond Gibbs in an important respect; j if P1 (fed claim) and P2 (state claim) sue D there is no supplemental jurisdiction j also if (P) sues D1 on a fed. claim and D2 on a state claim there is no supplemental jurisdiction. 1367(b) Hypo: P(Fl) sues D(Ga) & D(Fl) says he wants to join in federal court under Rule 20 j under traditional rule there would be no diversity jurisdiction because one P&D are from same state j exception to 1367(a) says that even if you are within 1367(a), 1367(b) makes clear that it supplemental jurisdiction does not apply to Rules 14,19,20,24 if 46

the plaintiff is the one bringing the claim. (it would lead to P circumventing the rule of complete diversity) j So long as you have complete diversity then as long as the jurisdictional amount is satisfied by one plaintiff, and second plaintiff can come under Fed. jurisdiction. 1367(c) are reasons that the courts may decline to exercise supplemental jurisdiction over a claim.

Jin v. Ministry of State Security Facts: 51 Falun Gong practitioners (Ps) who were visiting Chinese nationals, U.S. residents, or citizens, sued in the dist. court of Wash. D.C. a number of persons or entities associated with China, particularly the China Television Corp. (D). According to Ps, a narrative accompanied the footages which defamed certain Falun Gong practitioners living in the United States. Ps seeks injunctive and monetary relief. . D filed a motion to dismiss the defamation claim pursuant to Federal Rule 12(b)(1) and 12(b)(6).

Issues: (i) (ii) Can the court exercise supplemental jurisdiction over Ps state claim of defamation? Is the state claim barred by the statute of limitations?

Held: (i) yes. Ruble 12(b)(1) motion to dismiss denied. (ii) yes, Rule 12(b)(6) motion to dismiss granted. (plaintiffs win on jurisdictional grounds, but dismissed on 12(b)(6) grounds; this means they cant refile--- if it was found that there was no jurisdiction, they could have refilled in state court) j Concerning the issue of supplemental jurisdiction. o because the defamation claim arises under state law, Ds Rule 12(b)(1) jurisdictional challenge focuses on Ps assertion of supplemental jurisdiction pursuant to 28 U.S.C. 1367 o When a federal court has an independent basis for exercising federal jurisdiction, it may, in certain circumstances, also exercise supplemental jurisdiction over related state-law claims. o To determine when the assertion of supplemental jurisdiction is appropriate, the district court should apply a two-part test: (1) the court must first determine whether the state and federal claims derive from a common nucleus of operative fact; and (2) the court must decide whether

47

to exercise its discretion to assert jurisdiction over the state claim by considering whether judicial economy, convenience, and fairness to litigants weigh in favor of doing so. o In this case both sets of claims are factually linked in that they form the nucleus of an alleged overarching campaign to abridge or nullify Ps rights and liberties. The claims are so related that Ps would ordinarily be expected to try them all in one judicial proceeding. o Even if there are issues of complex law, thy are outweighed by the convenience of hearing them in one case. o A court may decline to exercise supplemental jurisdiction, however, if the state claim raised a novel or complex issue of state law, substantially predominates over federal claims, or remains after the court has dismissed a federal claim. j Concerning the statute of limitations o For a complaint to survive a Rule 12(b)(6) motion to dismiss, it need only provide a short and plain statement of the cliam and the grounds on which it rests. o However, a D may raise the affirmative defense of statute of limitations via a Rule 12(b)(6) motion when the facts that give rise to the defense are clear from the face of the complaint. 12 (b)(6) motion raises the validity of the complaint.

REMOVAL Most lawsuits, whether based on diversity or a federal question, can be heard in either state or federal court. In this instance, state and federal courts exercise concurrent jurisdiction. When such a case is filed in state court, the defendant has the option of having the case removed to the federal court pursuant to 28 U.S.C. 1441. The general rerquirements are: Removal is to the district in which the state action was filed: The law-suit must be removed to the district court of the district in which the state court is located [28 USC 1441(a)] For example, the (P) sues the (D) in a state court located in the same part of the state as the district court of the western district of the state. The action can only be removed to that district court, not to the district court of the eastern district.

The district court must have original jurisdiction: The lawsuit must be within the district courts diversity or federal question jurisdiction. [28 USC 1441(a)] Thus, if the plaintiff could have filed the lawsuit in federal court, the lawsuit is removable.

48

a) Removal in Diversity Cases. If the state action is within the federal courts diversity jurisdiction, all defendants must be noncitizens of the state. [28 USC 1441(b)]. The reason has to do with the major purpose of diversity jurisdiction: to provide a neutral forum for out-of-state parties. If a defendant is a citizen of the sate in which the federal court sits, the purpose for providing a federal forum in no longer relevant. b) Removal in federal question cases: If the lawsuit filed in state court arises under a federal statute or other federal law, removal is allowed only if such federal question appears in the plaintiffs complaint. The federal question may not arise from a defense the defendant makes. [28 USC 1441(b)] This rule of law is called the well-pleaded complaint. c) Removal jurisdiction not derivative of state court jurisdiction: Formerly, removal jurisdiction was viewed as only deriving from proper state court jurisdiction. Thus, if the state court lacked subject matter jurisdiction, the action could not be removed to federal court. The state action had to be dismissed and refilled in the proper state court before it could be removed. Since 1986, the federal court is permited to retain an action removed from state court even though the state court lacked jurisdiction over the calim [28 USC 1441(c)] d) Removal when lawsuits involve multiple claims: When a separate and independent claim that falls within federal question jurisdiction is joined with non-removable claims, the entire case may be removed to federal court, and the judge may remand all matter in which state law predominates. Questions on pg 212-213 #1. [look at 1441] a) b) yes, removal b/c it arises under the Constitution c) yes, based on diversity jurisdiction d) no, b/c if the (D) is a citizen where the case is pending in state court, it is not removabale, even though it could have been originally filed in federal court. (rational is that D wont be prejudice in his own state) e) yes, b/c now there is not only a diversity case, but you have a case arising under the Constitution f) no, no removal b/c even though there is complete diversity since there is one D that is a citizen where the case is filed there is no removal. #2. [look at 1446 & 1447]

49

a) plaintiff would allege diversity is present, and looking at 1446(b) the defendant is not citizen of the state where the action is pending. b) the case will not be removable, and 1446(a) says that since it is a submission to the court making certain representations than she will be subject to the penalties under rule 11 c) no, because the notice of removal of a civil action or proceeding shall be filed within thirty days after the receipt by the defendant, through service or otherwise, of a copy of the initial pleading setting forth the claim for relief upon which such action or proceeding is based, or within thirty days after the service of summons. d) yes, removable b/c if the case was not originally removable, then the service of the complaint does not start the 30 day period, it only starts when an amendment makes it removable. e) yes, b/c even if you ask for $50,000 in damages, btu show that you are entitled to $1 million, you are still entitled to $1 million in damages. If D can show damages might exceed $75,000 than it might be removable. f) 1446(b) sets a 1 year time limit on removal after amendment (starts at the time the complaint is filed) g) 1447

THE EERIE DOCTRINE


Look in Glannons 163-198 for summary Eerie and the Rules of Decision Act (RDA) require that in diversity cases a federal court must apply the law that would be applied by the courts of the state in which they sit. By the law, we mean court decisions as well as statutes. The federal courts are not free to decide for themselves the right rule of consideration, the duty a railroad owes to a trespasser, or the enforceability of exclusive contracts. Rather than create, general common law, their job in a diversity case is to apply state common law. The problem with Eerie is one of substance and procedure. While Eerie requires federal courts to apply state law to issues which there is no federal law-making power substantive issuesit was doubtful that the Court ever thought that Eeries command would require federal courts to follow state law on clearly procedural issues in diversity cases.

50

Before Eerie case there was Swift v. Tyson, the court said that federal courts do not have to apply the common law decisions of the state in a diversity case. Then there was Eerie (1938) which was the same year the Federal Rules of Civil Procedure were adopted (FRCP). Held that federal courts should apply state law in diversity cases. Conflicts arise out of these occurrences between substance/procedure. j Supreme Court in Eerie said it would address the distinction between local/federal law, and concluded that doctrine of Swift v. Tyson should be overruled. Ruled that it was unconstitutional under Swift to say that federal courts have no power to enact law applicable to diversity case. j Required courts in diversity cases to apply the common law of the state they are sitting in. TWO BIG QUESTIONS 1) How do you determine what the law of the several states is? a. if there is a SC case on point, you follow that b. if you encounter issues that have not been addressed by Sup. Court of state, in those situations the federal court asks, what would the SC do in this situation? i. consider SCs previous decisions ii. look at dicta of SC iii. lower courts/law journals 2) Line between substance and procedure. Federal Rules Enabling Act

2072. Rules of procedure and evidence; power to prescribe (a) The Supreme Court shall have the power to prescribe general rules of practice and procedure and rules of evidence for cases in the United States district courts (including proceedings before magistrate judges thereof) and courts of appeals. (b) Such rules shall not abridge, enlarge or modify any substantive right. All laws in conflict with such rules shall be of no further force or effect after such rules have taken effect. (c) Such rules may define when a ruling of a district court is final for the purposes of appeal under section 1291 of this title.

----From the case Guaranty Trust Co. v. York(1945): Addressed the question of what you do about statute of limitations. The plaintiff filed suit in federal court, that would have been barred in state court b/c it had already run under the state court statutes. Argument

51

was the federal court should ignore the state statute of limitations and follow the federal statute on this matter. First question was are statute of limitations substantive or procedural? --- they are really a combination of both. In this case, the court said that you dont ask those question. Really should ask, is the difference between the federal rule and the state rule outcome determinative? If it is outcome determinative the court must apply the state rule instead, to prevent diverse parties from gaining unfair advantages simply because they can choose federal court y If it is outcome determinative it is substantive. Statute of limitations is outcome determinative so the state statute should have been used because the federal statute was outcome determinative.

-----In Byrd v. Blue Ridge Electrical Co. it was reaffirmed that under York federal courts should apply outcome-determinative state law even on procedural issues as to which there is federal constitutional authority to make its own rule. Byrd changed the analysis a little bit though by saying that countervailing federal policies that arise from the courts status as an independent judicial system should also be considered. (Constitutional questions, etc.) y Byrd said that even if the difference between fed. and state was outcome determinative you would still follow federal rules if there was a strong federal interest involved.

Eerie Analysis After Byrd Nature of Issue Matters of clearly substantive law (bound up with the rights and obligatons created by state law) Matters of form and mode (procedure) where applying separate rule would likely affect outcome Matter of form and mode (procedure) where applying separate rule would likely affect outcome, but there are important federal countervailing considerations Matters of form and mode (procedure) where applying separate federal rule is unlikely to affect outcome Federal Court Should Apply state law (Eerie)

Apply state law (York)

Apply federal law (Byrd)

Apply federal law

52

Hanna v. Plummer (most important case for analysis) Says that there is 2 completely separate inquiries: 1. There is a federal enactment that is applicable (GO DOWN LEFT SIDE OF DIAGRAM) 2. There is not federal enactment that is applicable.
(GO DOWN RIGHT SIDE OF DIAGRAM)

y They both involve a question distinguishing between substance and procedure, these are two different ways of approaching it. y Part I and Part II as separated in the Glannon book 1. What if there is a federal rule or enactment and it is in conflict with state practice? y only question is, is the federal rule valid? (lenient standard applied; does it abridge or enlarge substantive rights? no, than its valid) o if its valid, even if its outcome determinative Federal Rules APPLY!!!!(main holding in Hanna) 2. If there is no federal enactment that is applicable, just a federal practice that fed. courts follow (federal judges commenting on evidence, jury instructions either oral or written) y if the difference between the state practice and the federal practice is outcome determinative you have to decide if federal interest outweighs state interest. If yes, use federal rule; if no, use state rules y Hanna modifies the outcome determinative test. You look at whether the choice of forum would affect the outcome at the outset of the case. If YES use State Rules Most difficult issue after Hanna: Is there a conflict between state practice and federal enactment? How do we know when there really is a conflict? y look at the specific requirements of the provisions, and consider what the purposes of the provisions are and how they seem to be intended to operate. Then see if there is a conflict on how the federal judge implements the provisions. Example: Alabama state law say if you lose appeal, defendant has to pay 10% of the judgment. The Fed. Rule of App. Proc. allows the federal court as a matter of discretion to make them pay % of judgment if the appeal is deemed to be frivolous. The court concluded that those two provisions are in direct conflict. IN that case, they used Federal Rules.

53

Example 2: Alabama courts do not enforce Forum Selection Clause, but Federal Courts have no such policy, and give deference to reasonable forum selection Clauses. Question is if there is a conflict between the two? Majority opinion said there was a conflict/ Scalia says there is no conflict, says the matter is merely an issue of state contract law (go down right side of chart) (dont forget to look if fed. interest outweighs state)

Supreme Court says there is no heightened pleading standard except for in Rule 9. Example: State rules says that the complaint must be pleaded with particularity. [go down the left side of the chart] is there a conflict between state and federal statute is it broad enough to cover the case yes is the federal rule valid yes THEN YOU APPLY FEDERAL LAW yes

Example: State law says in malpractice claims, 60 days after pleading, plaintiff has to file an affidavit with the doctor saying that injury occurred from negligent care. y whenever deciding if there is a conflict, have to look at the purpose of both statutes.

DAMAGES For pain and suffering damages (economic damages), there are state imposed caps. Liquidated damages are used when it would be difficult to prove what the damages actually are. Punitive damages are there to deter and punish egregious conduct. Only 6% of cases have an award of punitive damages and the median award is $50,000. Supreme Court has established standards for review to see if punitive damages awards violate due process. Specific relief- unlike damages, is an order of the court telling the party to do something.

PLEADING Rule 8 & 12 are main rules

54

What distinguishes common law pleading vs. modern pleading is that in modern pleasing it doesnt go back and forth. In modern pleading you just have a complaint and answer. Pg. 338 Common Law Pleading Demurrer (now 12(b)(6)) concede the truth of the opponents factual allegations but challenged their legal sufficiency. So What? Traverse (now denial under rule 8(b)) the opposite of demurrer, conceded the legal sufficiency of the plea, but denied its factual allegations. Not True Confession and Avoidance (now affirmative defense under rule 8(c)) concedes the legal sufficiency and the factual truth of the preceding pleas, but alleges additional facts that change their significance. Yes, but Old Common Law Pleading System

Plaintiff
Writ-declaration

Defendant
Demurrer 12(b)(6) (pg 337-338) If no demurrer than there is plea 1. traverse (denial) under Rule 8(b) 2. confession & avoidance (affirmative defense) Fed. Rule 8 (c)

Demurrer - also raises legal issues and terminates pleading or Traverse or Confession and avoidance

Under Federal Rule 7(a), there is just a complaint and answer, NOTHING MORE!!! y in the situation where there is no counterclaim, all you have is a complaint and an answer and that is all.

55

A complaint has to do two things y invoke a body of substantive law y sketch a factual scenario that, if shown ot be true, falls within that body of law. RULE 8 : GENERAL RULES OF PLEADING FRCP 8(a) Claims for Relief For Rule 8(a) there are 3 elements you need to do: 1. State grounds for jurisdiction y diversity y federal question y admirality y supplemental y personal 2. Short and plain statement of the claim 3. Make demand for judgment; have to identify damages sought y diversity cases must exceed $75,000 FRCP 8(b) Defenses; Forms of Denials A party may respond to a pleaders allegations with an admission, a specific denial, or a denial based upon insufficient information. Have 3 choices 1. admit 2. deny 3. deemed denied b/c the pleader lacks information to respond. FRCP 8(c) Affirmative Defenses An affirmative defense is any fact asserted by the respondent that vitiates the opposing partys claim. Rule 8(c) provides for the pleading of affirmative defenses and provides that misdesignated counterclaims will be deemed affirmative defenses. An affirmative defense is a defense that relies on factual issues not raised in the complaint. A defense is an affirmative one if the defendant bears the burden of proving it or if it does not controvert the plaintiffs proofs.

56

Affirmative defenses should be asserted in the answer or responsive pleading. Affimative defenses include; AOR, contributory negligence, duress, estoppel, fraud, illegality, laches, etc. All affirmative defenses are automatically denied as provided by rule 8(d) and require no response. FRCP 8(d) Effect of Failure to Deny If a response is required, a party failing to respond is deemed to have admitted all avertments, except the amount of damages. Conversely, if a response is not required, a party is deemed to have denied all avertments. [if there in an allegation or charge that requires response, a failure to respond is like admitting all the charges. on the other hand when a response is not required from an answer, a party is deemed to have denied all avertments] FRCP 8(e) Pleading to be Concise and Direct: Consistency Says parties should plead claims or defenses in a simple, direct, and concise manner consistent with the notice pleading standard fo Rule 8(a). RULE 12: DEFENSES AND OBJECTIONS MOTION FOR JUDGMENT ON THE PLEADINGS 12 a(1)A Says that defendant shall answer pleading within 20 days after being served with a summons or; 12-a(1)B- if waived within 60 days after the request of waiver was sent, or 90 days if the defendant is outside any U.S. judicial district. 12 b Every defense, in law or fact, to a claim for relief in any pleading, whether a claim, counterclaim, cross-claim, shall be asserted in the responsive pleading therto if one is required, except that the following defenses may at the option of the pleader be made by motion: (1) lack of subject matter jurisdiction (2) lack of personal jurisdiction (3) improper venue (4) insufficiency of process (5) insufficiency of service of process (6) failure to state a claim on which relief can be granted (7) failure to join a party under rule 19

y In common law pleading the entire purpose was to get to one issue, and you would go back and forth until you came to one issue. The Federal Rules deemphasize pleading, and emphasize non-technicality.

57

y Florida Rule 1-100(a) is a little different. If (D) raises an affirmative defense, and P wants to avoid it, then P has to enter a plea stating avoidance. It is not assumed.

y In pleading, you cant state a legal conclusions, have to specify how the Ds acts fell within the area of law you are bringing suit on. Have to give specific facts in a short and plain statement. Usually stating a legal conclusion will not get your case thrown out. y Two types of deficiency in a complaint that might lead to dismissal; might be a result of inept pleading, or it is factually insufficient. 1. insufficient specificity 2. failure to allege an element of the claim y if the court believes it is inept pleading, then the court will usually allow to fix it; but if its factually insufficient than the court might dismiss the case with prejudice. Equity
y system of law developed under the authority of the chancellor y allowed litigants who believed that law did not provide them with adequate

remedy to come under this court for issues of fairness y offered a different type of remedy from most courts y common law offered damages y other differences between common law courts and court of equity y pleading system at common law aimed at producing single issue y procedural rules y never tried before jury, even on questions of fact y parties could be required to testify
y y b.c equity can grant specific relief more easily than law lends it to hearing certain

types of cases more often, as in land cases


y also more fiduciary relationship cases b.c theyre affected by the personalities and

relationships involved and the court must be able to have parties testify to get this info and only in equity court can the parties be forced to testify y equitable defenses y estoppel y fraud y laches Federal Practice
y it is sufficient to simply alledge negligence y see federal form 9 in supplement

58

y defects: y insufficient specificity y failure to allege an element y both can be the result of inept pleading or factual insufficiency

Haddle v. Garrison Facts: Haddle (P) a former at-will employee of Healthmaster claims that he was improperly discharged by Garrison and Kelly (Ds) to deter his participation as a witness in a federal criminal trial. P seeks damages under 42 U.S.C. section 1985(2). Ds move to dismiss under rule 12(b)(6). Since 42 USC 1985 is a federal statute it comes under arising under jurisdiction in the federal court. In addition he is claiming violations of state Georgia law. --- Ends up coming under supplemental jurisdiction since the Georgia state claims come under the same transaction or occurrence as the federal claims. Defendant uses 12(b)(6) motion to try to dismiss for failure to state a claim. Says that since P was an at-will employee he could have been terminated at any time. Statute says that he has to be injured in his person or property and if he is terminable at will, he is not injured in his person or property. Source of (Ds) defense was that they followed Morast that held that an at-will employee has no property interest in his employment and, hence, has no cause of action under section 1985(2) if discharged from his job. Issue: Can P state a claim for damages by alleging that a conspiracy proscribed by section 1985 induced his employer to terminate his at-will employment? Lower Court Holding: The lower court dismisses the case on the authority of Morast. Since its a dismissal and the final judgment, he appeals the complaint. (if he wanted to amend, he would have to say he wasnt an at-will employee which he could not do truthfully consistent with Rule 11) Circuit Court Holding: Gives a four-line affirmation of trial court, without reference to holdings in other circuits. Supreme Court: Unlike appeal to 11th circuit, the appeal to SCOTUS is completely discretionary. SCOTUS grants cert. and overturns the Circuit Court. y The fact that employment at will is not property under Due Process Clasue does not mean that loss of at-will employment may not injure a person or his property under section 1985. y The sort of harm asserted by Pessentially third-party interference with at-will employment relationsstates a claim for relief under 1985, and such harm has long been compensable injury under law.

59

Consistency in Pleading p1 sues d1 & d2 What if p says that d1 ran the red light and d2 ran the red light. Is this permissible? y yes, a party may set forth two or more claims Rule 8(e)(2) Ethical Limitations RULE 11 Rule 11 can be viewed as a national ethical rule that imposes constraints on federal pleading. Rule 11 requires an attorneys or, if the party is not represented by an attorney, the partys signature on a complaint, motion, or any other paper filed or submitted to the court. Additionally, Rule 11 provides that the attorney has a duty prior to any filing with the court to not only conduct a reasonable factual investigation (which includes going beyond and testing his clients versoin of the facts), but also to person adequate legal research that confirms whether the theoretical underpinnings of the filings are warranted by existing law. Rule 11 can be violated by both the attorney and her client, but a client cannot be sanctioned for inadequate legal foundations. Rule 11 primarily imposes sanctions for failure to do adequate legal or factual research, as the following cases illustrate, but sanctions are NOT necessary. y Rule 11 does not apply to discovery, discovery documents are treated more rigorously. y Rule 11 is an objective standard reasonable attorney standard y Rule 11 recognizes that sometimes factual information will be limited without full access to materials needed. y Sanctions may be given, they dont have to. Represented party can never be handed out monetary sanctions. y If client gives false story (fraud) and the attorney to the best of his knowledge believes that the clients story is true, and files a claim what happens? o In this situation the attorney cannot be sanctioned. If the attorney did not violate the rule, than the party cannot be held for a violation. y 11(c)(1a) safe harbor provision if D wants to make Rule 11 motion, the proper way to do it is to serve the Rule 11 complaint, and not file it with the court for 21 days.

60

y The law firm can be sanctioned. y When the lawyer admits a mistake, the court will not likely impose monetary sanctions, but when a lawyer is notified of a mistake, and persists with the mistake, the court will likely give out sanctions. y Rule 11 does not apply to discovery; discovery rules have their own sanctions which are more strict than Rule 11.

Text of 11(b) Representations to Court. By presenting to the court (whether by signing, filing, submitting, or later advocating) a pleading, written motion, or other paper, an attorney or unrepresented party is certifying that to the best of the person's knowledge, information, and belief, formed after an inquiry reasonable under the circumstances,-(1) it is not being presented for any improper purpose, such as to harass or to cause unnecessary delay or needless increase in the cost of litigation; (2) the claims, defenses, and other legal contentions therein are warranted by existing law or by a nonfrivolous argument for the extension, modification, or reversal of existing law or the establishment of new law; (3) the allegations and other factual contentions have evidentiary support or, if specifically so identified, are likely to have evidentiary support after a reasonable opportunity for further investigation or discovery; and (if you dont have evidentiary support at first, it gives you an opportunity to provide evidentiary support for the claim after discovery) (4) the denials of factual contentions are warranted on the evidence or, if specifically so identified, are reasonably based on a lack of information or belief. Problems on page 355 1.(a) no sanctions, something has signed (b) it is not subject to sanctions under Rule 11, does not apply to discovery 2. lawyer can be sanctioned for not conducted an adequate factual investigation. the client could be held responsible for the violation to some degree. 3. No, nothing in the rule that says you need to amend a false position in a signed document, but if you keep advocating the position once it becomes clear that the claim is false, you can be sanctioned under Rule 11.

61

Walker v. Northwest Corp. Facts: Walker (P) and his attorney Massey filed a complaint against Norwest (D). The complaint stated that federal jurisdiction was based on diversity of citizenship. But the complaint on its face failed to allege complete diversity of citizenship and the factual allegations that tended to show the absence of complete diversity. The complaint said that (D) was a Minn. corporation but it did not allege the other defendants citizenships precisely stating that they were South Dakota residents. D informed Massey that there was no diversity jurisdiction and that Massey should dismiss the complaint or D would file a motion to dismiss and seek sanctions under Rule 11. Massey made no substantive response to D. The district court granted Ds Rule 12(b)(1) motion to dismiss for lack or jurisdiction and sanctioned Massey under Rule 11, ordering him to pay $4,000 in fees. Issue: Did the district Court abuse its discretion by imposing Rule 11 sanctions for failure to correct an obvious inadequate jurisdictional foundation in the complaint? Held: No. y It is the plaintiff's burden to plead the citizenship of the parties in attempting to invoke diversity jurisdiction. A plaintiff fails to carry that burden if they do not allege the domicile of the individual defendants or the place of incorporation and principal place of business of all the corporate defendants. y Federal diversity jurisdiction requires complete diversity, so that no defendant is a citizen of the same state as any plaintiff. y The Dist. Court did not abuse its discretion in awarding monetary sanctions rather than just dismissing the case. y The Dist. Court did not abuse its discretion in denying Ps request to amend their complaint, as Massey failed numerous times to allege the citizenship of many of the defendants. The district court is not obliged to do an attorneys research. Class notes: A federal court has jurisdiction to decide a case brought under a federal statute even if the claim is substantively insufficient.

Christian v. Mattel Facts: P sued Mattel (D) in federal court. The complaint, which Ps attorney (Hicks) signed, alleged that D obtained a copy of a copyrighted doll in 19996, and then developed a new Barbie doll which was substantially similar to the original. Despite evidence by D that the Barbie predatedPs doll, Ps attorney (Hicks) refused to dismiss the action. In response, D served Hicks with a motion for Rule 11 sanctions, arguing that Hicks had

62

signed a filed a frivolous complaint on a legally meritless theory that could be easily recognized with proper investigation. Hicks declined to withdraw the complaint during the 21-day safe harbor period provided by Rule 11. Court awarded Ds motion for summary judgment and Rule 11 sanctions on the fact that Hicks filed a friviolous complaint and further that he behaved boorishly during prior court proceedings. Issue: Did the court abuse its discretion in imposing Rule 11 sanctions? Held: Unclear, Rule 11 orders vacated and case remanded. y Court did find that attorneys complaint was frivolous. The evidence was clear that the Barbie doll was created six years before the Ps doll. y Although Rule 11 permits the court to sanction an attorney for conduct regarding pleadings, written motions, and other papers that have been signed and filed in a given case, it does not authorize sanctions for misconduct that occurred outside these areas, such as discovery abuses or misstatements made during the oral presentation. y Because it is not clear whether the court granted Ds Rule 11 motion as a result of an impermissible intertwining of its conclusion about the complaints frivolity and Hicks extrinsic misconduct, the courts Rule 11 orders are vacated and the case is remanded for further proceedings. y Can violate Rule 11 during oral argument if you are arguing a subject that is written in the signed papers. y Non Rule-11 Sanctions: (1) Court has inherent authority to impose sanctions for misconduct and (2) 28 U.S.C. 1927

SPECIAL CLAIMS Even if the pleading standards of Rule 11 have been satisfied, some claims have drawn special scrutiny from the courts. With respect to these disfavored claims, the general rule concerning federal pleading that pleading is adequate if it gives D fair notice of the claim (Rule 8a) is called off. Ps complaint must, therefore, do more than state simple conclusions. Federal rule 9 sets out some claims requiring such specific pleading or heightened pleading (e.g. capacity and fraud) as well as pleading requirements for each such claim. The U.S. supreme Court has issued rulings forbidding specificity in the pleading for claims based on 42 U.S.C. section 1983. Section 1983 claims allow a person granted a federally conferred right or entitlement to sue any person who, acting under color of state authority and for impermissible reasons, has prevented him from exercising such right or entitlement. Stradford v. Zurich

63

Facts: (P) sued insurers (D) in state court seeking payment on an insurance policy. Ds removed the case to federal court. Ds then asserted several counterclaims against P asserting that P knowingly and willfully devised a scheme to defraud Ds and obtain money by false representations. P moved to dismiss the counterclaims that are based on fraud for failure to state their claims with sufficient particularity under rule 9(b), and to dismiss certain other counterclaims under Rule 12(b)(6) for failure to state a claim. Ds then motioned for leave to amend their counterclaims pursuant to Rule 15(a). Issue: (i) Do Ds counterclaims assert the claim of fraud with sufficient particularly as required by Rule 9(b)? (ii) If not, should the court grand Ds Rule 15(a) motion for leave to amend the counterclaims to conform their pleading to Rule 9(b)? Held: (i) No. (ii) Yes. Motion granted. y Rule 9(b) provides, In all averments of fraud or mistake, the circumstances constituting fraud or mistake shall be stated with particularity. Malice, intent, knowledge, and other condition of mind of a person may be asserted generally. Here, Ds counterclaims succeed in alleging facts that give rise to a strong inference of fraudulent intent as required in the second sentence of Rule 9(b) y The counterclaims, however, do not satisfy the first sentence of Rule 9(b) which requires that the time, place, and nature of the alleged misrepresentations be disclosed to the party accused of fraud. Here, Ds counterclaims simply fail to identify the statement made by P that they claim to be false. It is unclear from the face of the counterclaims whether Ds assert that Ps claimed losses are improperly inflated, that {;s office never even flooded, or that the offices flooded, but not during the term of the insurance policy. y Additionally, the primary purpose of Rule 9(b) is to afford a litigant accused of fraud fair notice of the claim and the factual ground upon which it is based. Here, Ds counterclaims fail to prove P with fair notice of precisely which statement, or which aspect of his claim on the insurance policy, they allege to be false. The counterclaims are therefore insufficient under Rule 9(b), and must be dismissed. y Rule 15(a)provide that leave [to amend] shall be freely given when justice so requires. Therefore, Ds are granted leave to amend their counterclaims. y Even in civil rights claim, there is no heightened pleading standard in any situation other than fraud. Burdens of Pleading:

64

Lawyers usually distinguish among three burdens; the burden of pleading; the burden of production; and the burden of persuasion. (PLEADING) Burden of pleading means that one must allege that element of the claim or defense; one can not expect the other party to do so. (PROOF) To have the burden of production means that at trial one must produce the evidence to demonstrate the proposition at stake. To have the burden of persuasion in a civil case means, generally, that one must persuade the trier of fact that ones version of the facts is more likely than not to be true. Usually the three burdens go together. In an adversary system where we rely on the parties to take charge, BURDENS are important and can be decisive in some cases. y certain burdens are on the defense; statute of limitation, affirmative defenses Gomez v. Toledo Facts: Gomez (P) sued Toledo, the police chief of Puerto Rico (D), contending that D violated Ps constitutional due process rights by discharging P from his job as a police officer. P claimed that this action violated section 1983. First D denied the allegations in his answer, and stated a number of affirmative defenses, THEN moved to dismiss the claim for failure to state a cause of action 12(b)(6)--- (WRONG ORDERhave to move for dismissal before answer)The court granted Ds Rule 12(b)(6) motion on the ground that D was entitled to qualified immunity for acts done in good faith within the scope of his official duties and that P failed to allege that D lost his immunity by acting in bad faith. P appeals. Issue: Did P have the burden of pleading that D acted in bad faith when he discharged P? (Defendant is saying the burden of pleading was on the plaintiff) Held: No, judgment reversed. y Section 1983 only requires P to plead the deprivation of a constitutional or statutory right. y Qualified immunity is a common law doctrine that provided D with an affirmative defense against certain actions arising form the performance of an official duty. y Burden is on the defendant to plead good faith. because the D has a better opportunity show what his state of mind was, while the P cannot show that. y This is a fair allocation of the pleading burden because the qualified immunity defense frequently turns on factors that a plaintiff cannot reasonably be expected to know.

65

Hypo: Adams, a lawyer, sues his cousin, Davis, for professional services. Who, if anyone must allege that the services were intended as a gift? - - - since its a family member in this situation the burden will probably be on the plaintiff.

Responding to the Complaint.


Rule 55: Default Judgment: (a) Entry. When a party against whom a judgment for affirmative relief is sought has failed to plead or otherwise defend as provided by these rules and that fact is made to appear by affidavit or otherwise, the clerk shall enters the partys default. At common law there were 4 main ways a D could respond to a claim: 1. demurrer 2. dilatory pleas (jurisdictional) 3. pre-emptory pleas (traverse, confession and avoidance) Under Federal Rules (Defense) - failure to state a claim (demurrer) - lack of subject matter/ lack of pers. jurisdiction/ lack of venue/ insufficiency of process/ failure to join a necessary party (Similar to dilatory pleas under common law) - Denial (traverse) - Affirmative Defense (confession and avoidance) Procedural Devices to raise these defenses: Which defenses could be raised by motion? y 12(b) demurrers and dilatory pleas Which defense can be raised in an answer? y assuming that none of them have been waived, then ALL can be raised in the answer Why would you want to raise something in a motion if you could raise it in an answer? y you have more time (usually takes at least a month to rule on a motion; if the motion is denied than you get 10 days to provide an answer) y in the answer you have to raise everything (denials, affirmative defenses) if you really believe the complaint is insufficient, you dont want to have to go through the process of framing the answer and doing all the extra work if the complaint is insufficient. Every time limit (except commencement of case) is based on service. A defendant who fails to respond to a complaint can have a default judgment entered against her. For defendants who do want to respond, we grant two opportunities. The preanswer motion permits defendant to raise certain types of objections to the action at a very early stage of the litigation. If the defendant makes no such motion or if it is denied, the defendant must file an additional pleading, usually called an answer. The answer 66

responds to the allegations of the complaint and asserts any additional information or affirmative claims that the defendant may have against the plaintiff. Responses to a complaint include: (a) reasons why the court should not proceed with the action (b) assertions that the complaint, even if true, provides no legal basis for relief (c) denials (d) affirmative defenses and (e) requests for clarification and more information. y all the responses listed above except for (e) a request for clarification or more informationmay be included in the defendants answer. In addition Rule 12(b) permits certain defenses to be raised by a pre-answer motion. Rule 12(f) Motion to Strike Upon motion made by a party before responding to a pleading or, if no responsive pleading is permitted by these rules, upon motion made by a party within 20 days after the service of the pleading upon the party or upon the court's own initiative at any time, the court may order stricken from any pleading any insufficient defense or any redundant, immaterial, impertinent, or scandalous matter. Rule 12(g) Consolidation of Defenses in motions A party who makes a motion under this rule may join with it any other motions herein provided for and then available to the party. If a party makes a motion under this rule but omits therefrom any defense or objection then available to the party which this rule permits to be raised by motion, the party shall not thereafter make a motion based on the defense or objection so omitted, except a motion as provided in subdivision (h)(2) hereof on any of the grounds there stated. y only make one 12(g) motion and you cant raise any other motions is your defense (only 1 pre-answer motion) except for those in (h)(2) Rule 12(h) Waiver or Preservation of Defenses (1) A defense of lack of jurisdiction over the person, improper venue, insufficiency of process, or insufficiency of service of processis waived (A) if omitted from a motion in the circumstances described in subdivision (g), or (B) if it is neither made by motion under this rule nor included in a responsive pleading or an amendment thereof permitted by Rule 15(a) to be made as a matter of course. (2) A defense of failure to state a claim upon which relief can be granted, a defense of failure to join a party indispensable under Rule 19, and an objection of failure to state a legal defense to a claim may be made in any pleading permitted or ordered under Rule 7(a), or by motion for judgment on the pleadings, or at the trial on the merits. (3) Whenever it appears by suggestion of the parties or otherwise that the court lacks jurisdiction of the subject matter, the court shall dismiss the action. QUESTION ON PAGE 381 a. (1) no (2) yes

67

(3) no, need to consolidate everything (4) no, its waived. (5) yes, (6) yes, you can do it at any time b. (1) no, because it is waived if you dont include it a motion, nor in a responsive pleading. Have to include (h)(1) defenses in the FIRST thing you file or it is waived. (2) Since it cannot be waived he still can bring it up (3) no c. Look at Rule 12(d) ALWAYS INCLUDE DEFENSES IN FIRST THING YOU FILE!!!! DENIAL Rule 8 (b) (d) Rule 8(b) requires the defendant to deny only those allegations that he actually disputes, and Rule 8(d) provides that any allegation that is not denied is deemed admitted. Federal Rules say you are not supposed to generally deny everything, but do not prohibit it. Everything in Rule 8(b) encourages specific denials. Rule 8(b) language; Unless the pleaser intends in good faith to controvert all the averments of the preceding pleading, the pleader may (1) make denials as specific denials of designated avertments or paragraphs, or (2) may generally deny all the avertments except such designated avertments or paragraphs as the pleader expressly admits. Say there are 5 allegations. 1/2/3/4/5 y you only have to deny the ones you are denying, and the rest are deemed to be admitted. y If you want to deny 4 out of 5, you can have a general denial for the 4 and admit to the one. Zielinski v. Philadelphia Piers Facts: Plaintiff was injured when the forklift he was operating collided with another forklift. He alleged in a paragraph of his complaint that he was injured due to a collision with a forklift operated by an agent who was acting in the course of his employment with defendant. Defendant's answer contained a general denial of this paragraph, and in fact, an agent of a third party was operating the forklift at the time. However, the trial court ruled that defendant had admitted that the forklift operator was its own agent. Issue: Does a vague general denial of liability, where a specific denial of ownership was required, constitute bad faith pleading and give rise to an admission of Ds ownership

68

where P, in reliance on Ds misleading pleading, is precluded by the statue of limitations form suing the right owner? Holding: Since defendant did not contest plaintiff's injury or the fact of the collision, it should have made a specific denial only as to the agency allegation, and the general denial was thus ineffective. D has to admit that it was the employer even though it really wasnt because the general denial was ineffective. Also, defendant was not permitted to amend its answer to include a more specific denial, since the limitations period as to the third party had already passed. Class Notes: The lawyers failing to specifically deny that the employee was under their control made them fail to meet the pleading requirements under Rule8(b). General denial by Ds lead Ps to believe that Ds are denying responsibility for the negligence. Why did the Court think it was soo important for the defendant to admit this? y the Ps statute of limitations had ran up while the P was suing the wrong defendant, and then Ps would not be able to sue the right defendant. Ps can file an amendment to the original pleading under 15(c) to change the name of the defendant if it arises under the same transaction or occurrence so the statute of limitations does not run. Hypos pg. 387 a. Correct answer: admit (if you were B and did deny the allegation, during discovery and depositions the truth would come out) b. Correct answer: do not admit just because it was alleged, but you dont have any factual knowledge yourself, you dont have to admit use Rule 8(d) in this situation. c. Correct answer: do not admit --

Affirmative Defenses RULE 8 (c) An affirmative defense is the modern counterpart to the common law confession and avoidance. It may be defined as a defense in which the defendant, rather than denying the coplaints allegations, as in the case of a general or specific denial, confesses he truth of the complaints allegations but asserts that Ps theory of liability, even though sustained by evidence, does not apply. An affirmative defense must be presented in the answer or else it is waived and cannot be introduced at trial. Some affirmative defenses are listed in Rule 8(c)

69

In what circumstances can D use an affirmative defense not listed in 8(c)? y Dont want surprise in pleading Layman v. Southwestern Bell Telephone Facts: Layman (P) sued SW Bell (D) for trespass after Ds buried wires on her property. Ds pleaded only a general denial in response to Ps complaint. The case went to trial, at which time Ds introduced an exhibit showing that D had an easement to enter on Ps property and, hence, was not liable for trespass. P objected that the easement defense was inadmissible b/c it is an affirmative defense and thus should have been presented in the answer. The trial court overruled Ps objection and rendered judgment in Ds favor. Issue: Is a right of entry by easement an affirmative defense in an action for trespass? Held: Yes, judgment reversed. y Although Rule 55.08 (similar to 8(c)) which lists some affirmative defenses and specifies that such defenses shall be included in the answer, does not include an easement among the listed affirmative defenses, it is significant to note that a license, which is another form of authority to go upon land, is listed as an affirmative defense. y A general denial is not a way to raise the defense of easement. It would have been more informative for (P) to have raised it as a affirmative defense. y More important, it is clear that the defense of easement was used in this case as an attempt to avoid liability without denying any allegations in the complaint. That is exactly what an affirmative defense is. They should have denied all the other stuff, and just admitted to the trespass. Whats the Line Between a Denial and Affirmative Defense? REPLY Rule 7(a) Usually most pleadings end with the answer. Rule 7(a), however, requires the plaintiff to file a responsive pleading, which the rule calls a reply to a counterclaim. In addition to this mandatory reply, the rule allows the court to order replies on its own motion. Under FR a reply will almost never happen after an affirmative defense. Hypo #4 (pg 393) 12(f) move to strike motion/ but there is no need to assert a new pleading in response AMENDMENTS Rule 15(a)

70

Rule 15(a) allows a party to amend his pleadings one time as a matter of course (i.e. without permission from the court) (i) before a responsive pleading (e.g. an answer) is served or (ii) within 20 days after service of a pleading to which no responsive pleading is permitted (e.g. service of an answer containing a denial of affirmative defense), provided that the lawsuit has not been set fro trial. At all other times, a party must obtain permission from the court before it can amend its pleadings. Such permission shall be freely given when justice so requires, which means that as a practical matter it is incumbent on the party opposing the amendment to show good cause why the amendment should not be allowed. y *** If there has only been a motion, you can still have an automatic right to amend. But if there has already been an answer, then you have to ask the court for permission. (b) Says that when issues not brought up in the pleadings are tried at trial, and the P does not object, they will be tried if they had been raised in the pleadings. IF evidence is objected to at the trial on the ground that It is not within the issues made in the pleadings, the court may allow the pleadings to be amended and shall do so freely when the presentation of the merits are served. The court may grant a continuance to enable the objecting party to meets such evidence. (c) Relation Back of Amendments: An amendment of a pleading realtes back to the date of the original pleading when: (1) relation back is permitted by the law that provides the statute of limitaiotns applicable to the action, or (2) the claim or defense asserted in the amended pleading arose out of the conduct, transaction, or occurrence set forth or attempted to be set forth in the original pleading, or (3) can change the name of the party . (look up the rest.) Note: Rule 15says that leave to amend shall be freely given when justice so requires. the courts have read this phrase to mean: (a) that the would-be amendment should have a good reason for not getting the pleading right the first time; and (b) that allowing the change now shouldnt hurt the other side too much prejudice.

Beeck v. Aquaslide N Dive Facts: Beeck (P) sued Aquaslide (D) which P believed to be the manufacturer of a swimming pool slide on which P was severely injured. P claimed D was negligent, strictly liable, and in breach of an implied warranty. In its answer, D admitted that is was the manufacturer of the slide from the basis of an independent investigation of the slide by the insurance companies for D. Six months after the Stat. of Limits expired, Ds president, Meyer, visited the site of the accident and determined the slide was not manufactured by D. D moved for leave to amend its answer, to withdraw its admissions, and to have a separate trial on the manufacture issue. The trial court granted both

71

motions. The jury found that D was not the manufacturer of the slide, and the trial judge entered summary judgment in Ds favor. Issue: Did the trial court abuse its discretion by granting the motion for leave to amend the motion for a separate trial on the issue of who manufactured the slide? Held: No, judgment affirmed. As to the motion for leave to amend. y In the absence of any apparent or declared reason such as undue delay, bad faith or dilatory motive on the part of the movant, undue prejudice to the opposing party by virtue of allowance of the amendment, futility of amendment, refusal to eliminate deficiencies etc the leave sought should, as the rules require, be freely granted. Of course, the grant or denial of an opportunity to amend is within the discretion of the district court. y In this case there is no indication of bad faith, prejudice or undue delay. Ds admission was in reliance upon investigation by 3 different insurance companies. No prejudice to P resulted from the expiration of the statute of limitations, because P would not have prevailed on the merits of the manufacture issue. Furthermore, allowance of the amendment does not necessarily preclude P from proceeding against other parties. Finally, P had not at any time conceded that the slide in question had not been manufactured by the defendant. Notes: If a P wants to amend a complaint at the trial, the court will look disfavorable since there has been sufficient time to get it right (undue delay) It is the Plaintiffs burden to show that there is prejudice. Has to show evidence that a successful claim would be cut off. In this case, P would not have a successful claim against Aquaslide because they did not manufacture the product.

Statute of limitations and relation back: The Rules allow both Ps and Ds to amend their pleadings to add a new legal theory, a new party, or a new defense, whether or not such new matter arises out of the same factual pattern that forms the basis of the original complaint. Improper addition of new legal theories Moore v. Baker Facts: Moore (P) sued Baker (D) in federal court following surgery that went badly and left P severely disabled. P field suit on the last day of the statute of limitations. The initial complaint only alleged that D had violated a informed consent law. D filed a motion for summary judgment. P then moved to amend her complaint to assert a medical malpractice claim related to Ds performance of the surgery and post-operative care. The

72

district court denied the motion, stating that the medical malpractice claim was barred by the statue of limitations. P appeals. Issue: Does the proposed amendment asserting the medical malpractice claim relate back to the original complaint so as to save the medical malpractice claim from the statute of limitations? Held: No judgment affirmed. j A statute of limitations bars an amendment ot a claim initiated after the limitation period unless it relates back to the original complaint under Rule 15(c). j A claim relates back if it must arise from the same conduct, transaction or occurrence set forth in the original pleading and the original pleading must have given adequate notice to the defendant of the claim being asserted in the amended complaint. j The informed consent claim asserted Ps original complaint was a separate occurrence and, hence, did not put D on notice of a possible medical malpractice claim. The two claims were completely distinct in terms of time of occurrence, involved entirely separate facts, and involved separate and distinct conduct on the part of D. Proper Addition of New Legal Theories Bonerb v. Richard Caron Foundation Facts: Bonerb (P) sued Foundation (D) in fed. court for damages from slip-and-fall injury while playing on Ds bball court. Ps original complaint asserted negligence on the part of D in maintaining the court. P then filed a motion for substitution of counsel that was granted. P now wishes to amend the earlier complaint, using the relation back doctrine to overcome the expiration of the statute of limitations, to assert a claim of counseling malpractice based on info Ps new counsel claims to have uncovered. D objects. Issue: Does the proposed amendment asserting the counseling malpractice relate back to the original complaint? Held: Yes, motion granted. y In determining whether a claim related back and consequently should be allowed in violation fo the statute of limitations, the court must determine (1) whether the new claim arises from the same transaction or occurrence and; (2) whether the defendant was put on notice of the claim by the Ps original complaint.

73

y Absent undue prejudice to the party opposing the amendment, undue delay, or bad faith, the amendment should be granted if these two elements are in fact established. y The actual wording of Ps original complaint gives D a strong indication that the elements necessary to support the counseling malpractice claim are being asserted.

DISCOVERY
General Rule: 26 (b)(1)provides that parties may obtain discovery regarding any matter, not privileged, which is relevant to the subject matter involved in the pending action It is not ground for objection that the information sought will be inadmissible at trial if the information sought appears reasonably calculated to lead ot the discovery of admissible evidence. Rule 37 : Failure To Make or Cooperate in Discovery: Sanctions (look at 37 (2)(B) ) An order compelling discovery is NOT Appealable *** Only FINAL ORDERS are appealable. Relevance Davis v. Precoat Metals Facts: Black and Latino employees (Ps) sued their employer Precoat (D) alleging race discrimination. Ps allege to have been exposed to a hostile workplace, which included racially insulting comments by Ds management. Ps also claim that D discriminated against Black & Latino employees in terms of job placement and promotion, and that D retaliated and fired them after they complained. Ps requested a production of documents, but Ds refused saying information is not relevant and not worth the effort to produce. Ps filed a motion to compel discovery, seeking discrimination complaints made against he Ds by nonclerical/nonadministrative employees who worked at the same plant. Issue: Are Ps entitled to the requested discovery? Held: Yes, motion granted. y Rule 26(b)says that discovery is not limited to that which would be admissible at trial. Information relevant for the purposes of Rule 26 if the discovery appears reasonably calculated to lead to the discovery of admissible evidence. y However, a P is not necessarily entitled to all discovery that is relevant under the rule. Under Rule 26(b)(2) a court can limit discovery if it determines, among

74

other things, that the discovery is unreasonably cumulative or duplicative, obtainable form another source that is more convenient, less burdensome, or less expensive, or the burden or expense of the proposes discovery outweighs its likely benefit. Hence, the request of discoverable information must be reasonable under the circumstances. y Ds request for discovery was reasonable b/c it was limited to the same types of discrimination alleged in Ps complaint (racial, as supposed to age or sex discrimination) Thus, Ds arguments that Ps request are overly broad in that they improperly seek info regarding all allegedly discriminatory practices field against D is without merit. Ps requests are reasonable because they are narrowly tailored to their specific case.

Irrelevant Information Steffan v. Cheyney Facts: Steffan (P) sued Def. Sec. Cheyney (D) claiming that he was constructively discharged from the Naval Academy based on his statement that he is homosexual. Citing the constitutional privilege against self-incrimination, P refused the courts discovery order to answer deposition questions directed to whether P had engaged in homosexual conduct during or after his tenure as a midshipmen. As a consequence, the court dismissed Ps action under Fed. Rule 37(b)(2), which allows a court to sanction litigants for failure to comply with a courts discovery order. The court believed that question regarding Ps homosexual conduct were highly relevant, even though Ps discharge was nto based on any such conduct, because the Navy could refuse Ps requests for reinstatement on the ground that an individual has engaged in homosexual acts. P appeals. Issue: Did the court abuse its discretion in dismissing Ps lawsuit for failure ot comply with the discovery order? Held: Judgment reversed. y P was separated from the service based on his statement that he was a gay. He was not separated on the basis of alleged homosexual conduct. Therefore, questions concerning his gay conduct are irrelevant to the action pending before the court. y That P seeks reinstatement as a form of relief for an allegedly invalid separation does not put into issue the question whether he engaged in potentially disqualifying conduct unless such was a basis for Ps separation. Also, if P was discharged wrongfully, he has never been discharged in the eyes of the law. Class Notes: The issue of relevance is contigent on what the particular allegations of a case are, and the substantive law of a case.

75

PG 413 Hypos: 4(a): Not discoverable, because it has nothing to do with the merits of the case. 4(b): Yes, because how much punitive damage you pay is proportional to your wealth. Under Rule 26(D) says that Ps get to inspect any insurance agreement under which any person carrying insurance may be liable to satisfy all or part of a judgment. y courts say this promotes settlements. PRIVILEDGE:Rule 26 (B)(1) : Parties may obtain discovery regarding any matter, not privileged, that is relevant to the claim or defense of any party Upjohn Case: Says all employees of a corporation are protected under attorney-client privilege. How do you raise the issue of privilege or work product? y Rule 26 (B)(5) Example: If you have a letter and claim privilege, you have to say what it is about the letter that allows you to claim privilege. Mistakenly producing privilege ( pg. 406 of supplement) Seems to create a presumption that unintentional release of privileged material should immediately be sent back.

LOOK ON PAGE 130 of Supp. for Surveying Discovery


Mandatory Disclosures: Rule 26 26(a) Now rule specifically says that there is certain information you need to disclose right away without being asked. The old rule didnt have any mandatory disclosure. Now the name, address, and telephone # of each individual likely to have discoverable information that the disclosing party may use to support its claims or defenses, unless solely for impeachment. First you need to do mandatory disclosures, then you get to additional discovery. Hypos pg. 413 #5(a) : y Have to disclose witness since you will use him to support claim; 26(a)(1)(c) need to include any damage claims; y DONT have to disclose boss, because he is adverse to your claim. y His driving record does not need to be disclosed either since it is adverse. (b): Has to disclose mechanic information if there is reason that you might have to use that person then you have to disclose. Dont have to disclose boss, since he probably wont be used.

76

Rule 26(f): Conference of Parties; Planning of Discovery: The parties must as soon as practicable or 21 days before a scheduling conference is held, come together and confer to consider the nature and basis of their claims and defenses and the possibilities for a prompt settlement or resolution of the case, to make or arrange for disclosures required by Rule 26(a)(1) and to develop a proposes discovery plan that indicates the parties views and proposals concerning. Order of Case Service 120 days before 16 (b) scheduling meeting Appearance Conference of Parties 21 days before 16(b) scheduling meeting Disclosure 14 days after conference 16(b) scheduling meeting If you get additional information, you have a duty to supplement under 26(e) Interrogatories: Rule 33 written questions usually drafted by the lawyer, that are addressed to another party. Only available to be used against other parties. Probably best for basic factual information. With interrogatories you have the obligation to do a bit of research on the question being asked. Places a disproportionate burden on expense on the party answering the interrogatory. What is the function of 33(d)? y where the answer of the interrogatory is substantially the same for the burden party (D) and the (P)s, then it is ok for the burdened party to hand over the records to the Ps and tell them to get the information for themselves Hypo on 420: 1. Store doesnt need to answer because they are not a party, and interrogatories are not available to non-parties. You would have depose them with a subpoena. Manufacture does not have to respond if it is over 25 interrogatories (Rule 33(a)). y Is it a proper response for them not to answer? a. have to object, and answer to the extent they are not objectionable. if you decide not to answer at all look to Rule 37(d) deals with total failure to respond and addresses sanctions. Failure to act because there is a problem with thediscovery request is not enough to refuse to answer at all.

77

Depositions: Rules 28, 30, 31, 32 :One discovery device that can be used to gain information from non-parties. Have the advantage of not being limited to parties, and you can follow up questions. If the party giving notice of the deposition doesnt show up than the other party can recover expenses Rule 30(G)(1) If the party fails to serve a subpoena on the witness, and witness does not attend, that party has to pay expenses Rule 30(G)(2) If one side doesnt show up to the deposition, the attending party will be able to conduct the deposition, and the party failing to show up will not be able to. Cant do more than 10 depositions Rule 30(2)(A) Rule 30(B)(6) allows for a corporation being deposed to designate a person in the corporation to testify on matters relating to the organization. The persons so designated shall testify as to matter known or reasonably known or reasonably available to the organization. The corporation can set forth the matters on which the person will testify. If everybody shows up, and a third party witness doesnt show up, the witness can be held in contempt sanction subject to Rule 45 With a non-party you cant be forced to travel more than 100 miles - Rule 45(c) A party can be deposed anywhere. HYPOS: Page 420 #2. You can only instruct someone not to answer if its a privilege or to enforce a limitation instructed by the court. y Rule 30(d)(4) can be used to terminate the deposition (only use it if you feel the entire deposition is being used in an abusive way)not used very often y Rule 30(c) --The normal thing to do is that an objection is made and recorded and then the answer is taken subject to the objection. If it ever comes to the point where the evidence may be subject to trial, the court can decide on its admissibility. c) There are 3 ways it can go to the court y say that the time is up, and if you continue everything you say will be subject to objection y terminate the deposition y terminate the deposition, and then say that it is the fault of the other counsel for raising so many baseless objections

78

Inspection of Documents: Rule 34 Commonly used device for asking for production of documents that are crucial to the case. Also available for allowing entry onto land. Hypo page 423: 1. Yes, you have to produce documents that you will use in support of your defense, UNLESS it is solely for impeachment. There is certain types of evidence that is solely for impeachment (testifying purely based on a persons credibility). In this instance, you can certainly argue that this should be disclosed. y what if its supposed to be disclosed but it is not? Then 37(c)(1) says without a substantial justificationnot to disclose, unless it is harmless, will be sanctioned. a) not something that relates directly to the incident, so it will not have to be disclosed. a. if the photo is taken in direct anticipation of litigation by an agent of the party, then it can be withheld under the work product doctrine 2. Can subpoena under Rule 45 Physical and Mental Examination: Rule 35 Rules now authorize a physical or mental examination of a witness, unlike all the other discovery devices, this one requires (1) a court order and (2) good cause shown and notice to the person examined. y most normal case is someone who put his condition of controversy (auto accident, claim of emotional distress) court will order examination y but say that the party did not put himself in the condition of controversy (had a heart problem, eye sight problem, limitless possibility) the court imposed a limit on this by saying that if the other side wants to put your condition in issue they have to come forward with good cause that your physical condition attributed to the claim.

Request for Admission: Rule 36 What is the difference to answering a question in an interrogatory and answering the question in an request for admission? y Any matter admitted under Rule 36 means that it is stipulated as a fact of a case. It is a way to boil down what you obtained from discovery. It usually comes after you have had discovery to a certain extent, and then trying to re-enforce and narrow those issues. What if the party wants to not respond to the request for admission? y It is taken as an admittance. y The proper response if you dont want to admit something is to reject or deny it.

79

Suppose that the opposing party makes an objection to a response for admission that the requesting party thinks is wrong or insufficient? y The matter could be admitted on the spot, or if the court responds to a motion to the sufficiency they can force a better answer. Suppose the wrongful admittance gets past the court, and P supposedly proves the fact in question at trial? y 37(c)(2) says that if you fail to admit, and the other side is put to the expense of having to prove the issue, the other side can seek the cost of the motion, and the expense of proving that issue at trial. Question on 424 1. If its a core issue in the case, it is a possibility to request admittance, and if it is denied falsely than it is possible to get fee shifting.

Ensuring Compliance: Rule 26(g) and 37 One step vs. Two-Step Sanctions y when a party fails to respond to depositions, interrogatories; can be subject to 30(b)(2) sanctions; the more common procedure when a party doesnt answer, would be for the other party to move the compel the answer under 37(a). If the court forces the party to provide an answer, and they still dont then 30(b)(2) motion to dismiss sanction can be levied. (Two-step) 26(g) is more stringent than Rule 11 because; - it makes sanctions mandatory - sanctions are more strict Rule 37 is the general provision for different types of sanction 37(d) one step sanction 37(b)(2) two-step sanctions 37(b)(2)(a): Failure to comply with order: one of the sanctions is that a fact can be ordered established 37(b)(2)(c): Case can be dismissed. Question 425 1a. What is the problem with taking depositions before the 26(f) conference? Unless there is something unusual you cant seek the disclosures of a party before the 26(f) meeting. 1b. Is it ever the correct procedure just not to show up to depositions?

80

j No, need to show up regardless. If think it is the best idea to postpone the deposition, can file a 26(c) motion and get a protective order. 26(c) can alter the sequence of discovery. j If things are working smoothly this issue should have been addressed at the 26(f) meeting, and then included in the scheduling order under 16(b) 2. The supplier did not have any notice of this, so under 37(c)(1) if you fail to produce evidence that you are going to use, you wont be allowed ot use it. 3. 26(g)(2) says every response, request, or objection, must be signed by a party which works as a certification, and it should be to the best of the Ds knowledge the information they are providing is correct. Rule 33(b)(1) says that if you answer an interrogatory falsely under oath then a sanction could be levied, and under 26(g)(2) that sanction would be attorney fees. 4a. BEFORE YOU MAKE ANY MOTION YOU ARE SUPPOSED TO CONFER WITH OTHER SIDE!!! BUT. if you have done this, and the other party fails to comply you can make a motion to compel under 37(d). 4b. One way would be to move for a protective order, and assert privilege as grounds for the protective order. A claim of privilege has to be expressly statedRule 26(b)(5) OR you can object with a Rule 34 request to object to a request for a production or interrogatory. Probably the preferred avenue, if it is something very egregious, might be a better idea to go with the protective order.

DISCOVERY AND PRIVACY


Intro: Discovery conflicts with the privacy preferences of the individual. Foreign lawyers find this aspect of U.S. litigation distasteful. Courts are often asked to weigh these conflicting interests. General Problem of privacy. Rule 26(c) Protective Orders Two ways issue can be 1) party seeking discovery moves to compel production 2) party who is objecting to discovery, moves for a motion to protect 26(c) says that before P can go to the court to compel discovery, need certification that the P has in good faith conferred or attempted to confer with the other parites in an effort to resolve the dispute without court action. If you are the one who necessitates the issue being brought to court, and you lose, you can be required to pay expenses. 2 main functions of protective order:

81

1) some issues you cant discover at all 2) other things can be discovered, but only within the scope of the litigation Stalnaker v. Kmart Corp. Facts: Stalnaker (P) sued Kmart (D) in court for sexual harassment, for inability to control its employee, Graves. P noticed the depositions of four female employees of D who may have been victims of sexual harassment at Ds place of employment. D filed a motion for a protective order pursuant to Rule 26(c) protecting these nonparty witnesses from discovery concerning voluntary romantic conduct or their sexual activities. Issues: (1) Should the protective order be issued? (2) Is info concerning voluntary sexual activitiy of nonparty witnesses barred from discovery under Fed. Rule of Evid. 412? Held: (1) Yes in party (2) No. y Rule 26(c) provides that the court, upon showing of good cause, may make any order which justice requires to protect a party or person from annoyance, embarrassment, oppression, or undue burden or expense. Also, a party is entitled to request a protective order to preclude anyh inquiry into areas clearly outside the scope of appropriate discovery. y To reduce the potential of embarrassment or annoyance to deponents and D, the parties may not use discovered information outside the lawsuit. y Even when a motion for a protective order under Rule 26(c) is made, the information may be barred under FRE 412(a). which aims to safeguard the alleged victim against sexual stereotyping that is associated with public disclosure of intimate details. However, Rule 412 does not apply unless the person against whom the evidence is offered can reasonably be characterized as a victim of alleged sexual misconduct. Thus, the rule does not control in this case. Comment: The magistrate was concerned that P might misuse the discovered information; hence, the imposed limitations. Notice that D did not seek a protective order for all romantic and sexual activities. This is because involuntary act ivies constitute sexual harassment. DISCOVERY IN AN ADVERSARY SYSTEM WORK PRODUCT Privilege and Trial Preparation Material. Hickman v. Taylor Facts: Hickman (P) brought a wrongful death action against Taylor (D) for the death of her husband, who sank on a tug ship. Ds attorney (Fortenbraugh), in anticipation of a lawsuit, interviews various persons with information about the case and collected info in two broad forums: (1) written and signed statements from survivors; (2) private memos

82

of oral interviews with witnesses, and mental notes. P sought discovery of each category of information and Ds attorney objected on the ground that the info was privileged. Issue: Is the information prepared by Ds attorney in anticipation of trial privileged and hence protected from discovery? Rule: "Where relevant and non-privileged facts remain hidden in an attorney's file and where production of those facts is essential to the preparation of one's case, discovery may be properly had. Such written statements and documents might, under certain circumstances, be admissible in evidence or give clues as to the existence or location of relevant facts. Or they might be useful for the purposes of impeachment or corroboration. And production might be justified where the witnesses are no longer available or can be reached only with difficulty." Held: Yes, judgment affirmed. y The information sought is relevant and not within the attorney-client privilege because it does not involve a communication between an attorney and his client. y An interrogatory is not an appropriate way to get information for a non-party, or get documents (from lawyer), interrogatories are used to ask questions, not get documents. y A conditional privilege for an attorneys work product was fashioned. Accordingly, all relevant and non-privileged materials prepared by an attorney with an eye towards litigation are free form discovery unless the party seeking discovery can show (i) a substantial need for the materials (ii) an inability to obtain equivalent material by other means. y The written statements and private memos contained info that is readily available to P because the witnesses to the sinking are available and not difficult to reach. Although oral statements made by witnesses to Ds attorney may not be readily available to P, a showing of necessity cannot be made with respect to them in this case, because forcing Ds attorney to write down his mental impression of the witnesses remarks gives rise to the dangers of inaccuracy and untrustworthiness and would not qualify as admissible evidence. y Written statements prepared in the context of litigation, and the memoranda based on witness interviews are protected under work product because it might contain opinions and trial strategies. Court says that other party should not be able to freeride off the hard work of the other attorneys. Only a showing of a substantial need or necessity would allow other party to discover. Work product doesnt pertain to the underlying facts. y Core Work Product: Mental impressions, conclusions, opinions, or legal theories of an attorney or other representative cab never be discovered.

83

Courts Rationale: Discovery has a two-fold purpose: (1) to narrow basic issues between parties; and (2) as a device for ascertaining the facts, or information as to the existence or whereabouts of facts, related to those issues. The deposition-discovery rules are to be accorded a broad and liberal treatment, but there are limits (oppressing the other party, bad faith). Memoranda, statements and mental impressions in issue in this case fall outside the scope of the attorney-client privilege and thus aren't protected from discovery on that basis. Nor is there privilege for information which counsel secured from a witness while acting for his client in anticipation of litigation, and the privilege does not apply to anything prepared by counsel for his own use in prosecuting his client's case, or to writings which reflect an attorney's mental impressions, conclusions, opinions, or legal theories. Why Have Work product? y lawyers free to write opinions, thoughts without worry of turning them over y opposing party can try to call the lawyer to testify to what the witness had told them, which would lead to the role of a lawyer as a witness which would damage their role as an advocate. Comment: The Courts opinion has useful commentary on the proper use of discovery. The court notes that Ps use of interrogatories and request for documents pursuant to Rules 33 & 34, respectively, were improper because these rules only apply to a party; and Ds attorney, against whom such discovery was sought, obviously was not a party. The proper procedure would have been to take Ds attorneys deposition and then to force production of the documents by serving a subpoena duces tecum pursuant to Rule 45. Rule 26(b)(3) expands Hickman work product doctrine by (i) giving the qualified privilege to a party or his representative which includes not only his attorney but investigators or others who might work for the party, and(ii) giving an absolute privilege from discovery to mental impression, conclusions, or opinions of a partys attorney or other representative. Pg 444 3a. First question is: Is it work product? (Is it prepared in anticipation of litigation?) here it is not necessarily the case, because accidents are routine. b. name of eyewitness must be turned over since its not work product. 4a. (a)if there is a specific request for this kind of information, the proper procedure is to indentify it, then claim the privilege (b) yes, 26(b)(3)(2nd paragraph) 5.a This would meet the need for unavailablility b. If the memory has faded, if might fall under unavailability and may be discoverable. c. Both would come under the general category of work product; should they still be discoverable? Film--- Defense would want it to be shown at trial for shock, powerfulness of video, Plaintiffs would not want this to happen because the video may be skewed in the Defense favor. One way to settle this is to depose plaintiff before video is revealed, and get their view on it, before it ever comes out.

84

Expert Information: Two provisions in Rule 26 govern the discovery of expert


information. Judge has to decide whether proposed expert testimony meets these threshold conditions. a. Mandatory Disclosure: As a part of mandatory disclosure, Rule 26(a)(2) requires the exchange of expert information: the identity of testifying and nontestifying experts; the basis for their testimony; and a written report prepared and signed by the witness... containing a complete statement of all opinions to be expressed and reasons therefore.... b. Additional Discovery: Rule (26)(b)(4) contains special provisions for additional discovery of experts. The rule attempts to balance two competing procedural policies: a policy against freeloading and laziness by one party who seeks to develop his case through the efforts of another party, and the basic policy that underpins all discoverynamely, that both parties should have access to all vital information prior to trial. The following cases deal with the protection Rule 26(b)(4) affords to the opinions of nontestifying parties. Experts can testify to their opinion, they are brining some sort of specialized knowledge that it is out of the realm of the jury. 1) sufficient facts or data 2) applied facts or data to case 3) whether theory or technique can be tested, subjected to peer review, whether they know the error offered to the jury. Motion in limiting requires the court to decide ahead of time whether evidence meets the conditions for admissibility. 4 provisions of Rule 26 which have a bearing on this: 26(a)(1): General obligations to make mandatory disclosure 26(a)(2): Expert Disclosures: Experts can testify to hypotheticals, or can apply special knowledge to facts. (b)(2) expert becomes part of team of party this rule is very detailed as to the fact needs to report all opinions and the basis and reasons behind them, the data or other information considered by the expert in formulating their opinion; the qualifications of the witness, etc. (normally they would not be discoverable because it would be core work product) 26(b)(1): General Scope of Discovery 26(b)(4): Further discovery in regard to Expert Testimony: Party may depose any expert, retained or unretained, but if its a retained expert then the deposition will occur after the report is given under 26(b)(2). In part 26(b)(4)(b) it talks about experts who you retain and dont want to testify. You might retain a doctor or economist and dont

85

want them to testify; you only want to consult them with the benefit of formulating the case. Rule protects disclosure of these consulting experts. Work Product, EXCEPT is goes to FACTS as well as opinion. All experts who may be used to give expert testimony to testify Under 26(a)(2)(A) all the experts who give expert testimony must be disclosed. -Can dispose experts who plan to give expert testimony under 26(b)(4)(A) Experts who are retained to testify: Have to give full report, in addition to identifying them under 26(a)(2)(B) ; after you got the report you can depose them under 26(b)(4)(A) Retained experts who are not going to testify: If they are experts who are not going to testify, then they do notneed to be disclosed; 26(b)(4)(B) they are immune from discovery, unless you can overcome the protection if there are exceptional circumstances.

Hypo on page 447: 2. (1) the names of the doctors (2) Dr. Welby needs to disclose his report, Dr. Kildare, not retained, does not need to provide any report. y Can both be deposed. b. Need not identify him, because he is not going to be called. c. Can be deposed before anything before he was retained, but after retention Welby under 26(b)(4)(B) Thompson v. The Haskell Co. Facts: P commence an action against Haskell (D) alleging that one of the workers sexually harassed her, and when she refused to cooperate she was fired, and as a result suffered severe depression. Ten days after she was fired, a psychologist retained by Ps prior to cousel performed an evaluation of P and then gave the report to Ps attorney. As a part of its discovery, D sought access to this report. P now moves for a protective order to shield the report from discovery pursuant to FRCP 26(b)(4)(B). Issue: Does FRCP 26(b)(4) exempt from discovery a report prepared by a nontestifying psychologist retained by Ps prior attorney when no other report of comparable value exists? Held: No, motion denied/ y Rule 26(b)(4) permits a party to discover facts known or opinions held by a nontestifying expert who was retained by another party only when Rule 35(b)

86

permits such discovery or when there are exceptional circumstances making it impracticable for the party seeking discovery to obtain facts or opinions on the same subject by other means. y If there are exceptional circumstances favoring disclosure then it might have to be disclosed. y Since the report evaluated Ps mental and emotional state only 10 days after her termination, it was impossible for D to obtain a comparable report during the following weeks after her discharge. Comment: D did not have an equal opportunity to subject P to a psychological evaluation immediately following her dischargeD may not have suspected that shortly after it would be engaged in litigation with P, and therefore did not recognize the need for obtaining a report comparable to that prepared by Ps expert. This alone may constitute an exceptional circumstance justifying disclosure. Chiquita International v. Bolero Reefer Facts: (P) Chiquita claimed it hired Bolero (D) to transport goods to Germany, but due to complications not all the goods got loaded. When the cargo arrived in Germany, it was incomplete and in poor condition. P contracted Winer, a marine surveyor, to examine the vessel and the loading gear. D sought discovery of Winers findings via a deposition and access to a fiel complied in connection with his inspection. P opposed this request on the basis that 26(b)(4) shields a nontestifying expert from discovery. D now seeks a motion to compel discovery of Winer on grounds that he is not an expert, but rather a fact witness and in the event that Winer does qualify as an expert, exceptional circumstances exist that justify discovery. Issues: (i) Does a nontestifying surveyor hired by a party to inspect a vessel involved in litigation classify as an expert for purposes of Rule 26(b)(4)? (ii) If so, does the exceptional circumstance clause of 26(b)(4) apply when that surveyor was the only expert who inspected the vessel shortly after it had arrived at its port of destination? Held: (i)Yes (ii) No. Motion denied. y Generally, Rule 26(b)(4) restricts discovery of facts known or opinions held by a non-testifying expert. y An expert does not lose his status merely because he has learned facts as a result of conducting personal investigation. y Rule 26(b)(4) contains an exception permitting discovery of facts known or opinions held by a non-testifying expert when exceptional circumstances exists that warrant a disclosure. y The exceptional circumstance clause does not justify disclosure when a party was not barred from obtaining the same information through its own source but failed

87

to do so. Allowing discovery under such circumstances would permit the exceptional circumstances to swallow the rule. y In this case D was not precluded in any way from sending its own expert ot inspect the vessel. At the very least, D had equal opportunity to conduct a timely inspection. Ds failure to do so should not be rewarded by permitting discovery of Ps expert. y Since Rule 26(b)(4) applies to document discovery as well as to depositions, D is barred from deposing Winer as well as having access to those portions of his file that contain his record. Comment: This case Thomspon are similar in that the information sought in both cases was irreplicable at the time discovery was sought. But discovery was denied here because D was negligent in obtaining the information sought-D could have obtained the information had it acted diligently.

RESOLUTION WITHOUT TRIAL


y Throughout Rule 16(b) conference, the subject is settlement is often discussed. y Court ordered mediation can bring the parties to the point of settlement Default judgment under RULE 55 (when defendants fails to file a timely answer to a properly served complaint and summons) is a pretrial termination of a lawsuit on procedural grounds. Default judgment is disfavored, because judges believe that cases should be tried on the merits. y Court would frown on it, if P sought a default judgment at the second the 20 day time period to answer has lapsed. Courts will allow D to vacate default judgment if done immediately. Two events: (a) Entry: After you file for default judgment, then the clerk enters it into the docket. Indication that the default has occurred (b) Judgment: (1) By the Clerk: The clerk can enter the default when it is for a certain sum of money; one limitation is if the defendant has appeared in the case (any action taken in the case that indicates the defendant will put on a defense) (2) By the Court: All other cases, where there is liquidated damages, etc. Rule 54(c) in a default judgment, you can only get what you asked for in your prayer for relief; it is a important limitation, because the party cant introduce evidence to get more damages. If the D thinks the amount P asks for is reasonable, the D may simply default and pay this amount.

Peralta v. Heights Medical 88

Issue: Must a default judgment that was entered without proper notice to D stand in the absence of Ds showing of a meritorious defense? Held: No, judgment reversed y A judgment entered without proper service is void y Even though D may lack a meritorious defense on the merits, the default judgment prejudices D because if D had notice of the lawsuit he could have done a number of things to protect his property from the sheriffs sale, or to otherwise avoid default judgment. Failure to Prosecute: Involuntary Dismissal: Like the default judgment, involuntary dismissal Rule 41(b) (e.g. for the plaintiffs failure to prosecute the lawsuit or for failure to comply with court order) is a pretrial termination of a lawsuit on procedural grounds rather than on the merits. Voluntary Dismissal: Rule 41(a)(1) gives the plaintiff the right to voluntarily dismiss her action at any time befor the defendant answer or files a motin for summary judgment. Usually, the dismissal is without prejudice. By limiting the plaintiffs dismissal right to an early state of the proceedings, Rule 41(a)(1) attempts to safeguard the plaintiffs right to terminate the proceedings whiel at the same time preventing arbitrary dismissals after an advanced stage of the proceedings has been reached. Rule 41(a)(2) gives the plaintiff the opportunity to dismiss her lawsuit beyond the early stage of the litigation only with the court s opinion. (if a dismissal has been asked for after a favorable ruling for the defendant, then the court is less likely to grant the dismissal). Because of the timing of the dismissal and the possibility that the plaintiff may restart her lawsuit at a later time, there is a possibility that dismissal would prejudice the defendant. SUMMARY JUDGMENT: RULE 56 The Concept: Summary judgment is a procedural device that provides for a summary disposition ofa n action where, on the undisputed facts, the movant is entitled to judgment as a matter of law. Summary judgment functions to bypass the plenary trial on the merits. Summary judgment is reserved for those relatively few instances where the fact resolving abilities of the fact finder are not necessary. Requirements: A motion for summary judgment can be granted only if 2 requirements are met: (1) The admissible evidence before the court must show that there is no genuine issue of material fact. (2) The movant must be entitled to judgment as a matter of law on those undisputed facts.

89

States of the evidence 1. between 10%-30% chance that the event occurred (judge) 2. between 30%-60% chance that the event occurred (jury) 3. between 60%-100% chance that the event occurred (judge) Reasoning is that unless there is sufficient evidence that the law says is sufficiently needed for the case, it should not go to the jury because the jury can be irrational. Most effective device from keeping a baseless claim away from the jury. [FRCP 56] The first requirement is at issue in most of the litigation involving summary judgment, as the following case illustrates. Hypo page 513: 1. a. Denied because there is already a claim, and a 12(b)(6) motion is for failure to state a claim. Have to apply the small amount of facts to the law, to make sure it is a legal claim. 12(b)(6) is just if the complaint is sufficient (Ps could still lose the case on the evidence) 2. What goes on from there? Discovery, Trial. What if the plaintiff introduces the note and shows that the plaintiff never paid and the D does not do anything to defend this? y P make a motion for a directed verdict. y what if at the trial the plaintiff introduce the note and says that it hasnt been paid, but the D shows evidence that the check has been paid? o D motions for a directed verdict for  at a trial where the D/P does not respond, then it wont go to the jury This is what happens at Summary Judgment BEFORE THE TRIAL, when there isnt sufficient evidence to justify a case going to trial or a jury. Standards for Determining Presence of a Triable Factual Issue: Once the moving party has carried its burden under Rule 56(c) of demonstrating the absence of a genuine issue of material fact, the party opposing the motion must come forward with specific facts showing that there is a genuine issue for trial (Rule 56(e)). Burden of evidence (production) means that party has to put in enough evidence to get to State 2 and get to trial. Hypo: y Say the plaintiff puts in evidence at trial and we wind up in State 1 the D would move for directed verdict. y Same, but now P puts in evidence in State 2, then D could introduce additional evidence that casts doubt on the Ps evidence that can push it back to State 1.

90

y Say now it ends up in State 3. P can move for directed verdict if D has nothing else to offer, and it would go to judge. When we wind up in State 3 plaintiff has shifted burden of production, and D has to put evidence to move it back into state Look at Houchins case in the beginning. [P was unable to carry burden of production and D moved from summary judgment and won]

Celotex Corp. v. Catrett Facts:P filed wrongful death against D and several other corps. claiming her husbands death resulted from exposure to asbestos manufactured by the corporation. After one year of discovery, D moved for summary judgment on the ground that during discovery P failed to produce any evidence to support her allegation that husband had been exposed. In response, P produced documents showing causation. D argued that were inadmissible hearsay and thus could not be considered in opposition to the summary judgment motion. Agreeing with D, the court granted Ds motion. The appellate court reversed on the ground that D failed to meet its initial burden of production under Rule 56(C)of showing the absence of a genuine issue of material fact, because D made no effort to adduce any evidence, in the form of affidavits or otherwise, to support its motion. D appeals. Issue: If information derived through discovery indicates that the nonmoving party (here P) cannot prove an essential element of her cause of action at trial, must the moving party (here D) still come forward with evidence in the form of affidavits or other information to show the absence of a genuine issue of material fact within the meaning of 56(c)? Rule: Where the P has the burden of production at trial, on a motion fro a summary judgment, all the defendant has to do is point out that the P has not evidence to support that motion. y what do you need to do to say that the P lacks the necessary evidence? o can use an interrogatory, and under 33(c) it says you can use an interrogatory where the answer to the interrogatory involves an opinion or contention that relates to fact or the application of law to fact. P, in response has to come up with some specific facts, to prove that there is a genuine issue of material fact. Cant rely on allegations, need facts. Held: No, judgment reversed. y There can be no genuine issue of material fact when, after an adequate discovery period, the party opposing summary judgment fails to make a showing sufficient to establish the existence of an essential element in that partys case, and on which that party will bear the burden of proof at trial. y The burden is not on the party moving for summary judgment to produce evidence showing the absence of a genuine issue of material fact with respect to

91

an issue on which the nonmoving party bears the burden of proof. Rather, in this situation, the movant need only show, that this burden may be discharged by relying on depositions, answers to interrogatories, and the like, as Rule 56(c) clearly provides. y The standard governing the burden of production for summary judgment mirrors the standard for a directed verdict under Rule 50(a), in which D would not be required to support its motion with affidavits or other similar materials negating Ps claim. The burden in on P, not D, to go forward with evidence as to Ps claim. Bias v. Advantage Intl Facts: The Estate of Bias (P) sued Intl (D) in court for injuries allegedly arising out of a representation agreement between Bias and Ds. This agreement, which Bias entered into after the close of his basketball career, called for Ds to advise and represent Bias in his affairs and to secure a $1 million life insurance policy on Biass life. Ds failed to secure such a policy despite assurances that they had done so. The court awarded summary judgment in favor of Ds pursuant to Rule 56(c). P appeals. Issue: Was there a triable issue of fact as to whether Bias was ever a drug user? Held: No, judgment affirmed. y Supreme Court explained that summary judgment is appropriate, no matter which party is the moving party, where a party fails to make a showing sufficient to establish the existence of an element essential to that partys case, and on which that party will bear the burden of proof at trial. Thus the moving party must explain its reasons for concluding that the record does not reveal any genuine issue of material fact, and must make a showing supporting its claims insofar as those claims involve issues on which it will bear the burden at trial. y Rule 56(c) provides for summary judgment where the pleadings, depositons, interrogatory answers, and admissions on file, together with affidavits, if any, show that there is no genuine issue as to any material fact and that the moving party is entitled to a judgment as a matter of law. y Thus a moving party must explain its reasons for concluding that the record does not reveal any genuine issues on which it will bear the burden at trial. y Once the moving party has carried its burden, the responsibility then shifts to the nonmoving party to show that there is, a genuine issue of material fact. The nonmoving party must come forward with specific facts showing that there is a genuine issue for trial. y Applying to the facts: (1) no genuine issue as to the fact that Bias was a drug user. Ps did not rebut former teammates testimony about Bias prior drug use (thus

92

failing to cast more than speculative doubt on the credibility of that testimony). therefore, the court properly concluded that there was no genuine issue of fact concerning Biass status as a cocaine user. y (2)Bias could not have obtained jumbo insurance policy because he was a drug user. Comment: Had P deposed former teammates, who might not have been credible witnesses if they used drugs with Bias, an issue of credibility could possibly have been raised. Ds evidence would not have been credible and, hence, sufficient to withstand the nonmovants attempt to show the existence of a genuine issue of material fact.

THE RIGHT TO A JURY TRIAL Key thing we are concerned about in trial is who decides the case? FRCP 38 is an endorsement for the right to a jury trial, but in FL you need to demand the right to a jury trial in Federal Court. Background: The 7th amendment to the Const. establishes the right to a jury trial: Based on the language that the right to trial by jury shall be preserved... according to the rules of common law, courts have adopted the historical test for determining whether the right to a jury trial exists in a given case. Specifically, parties have the right to trial by jury if they had that right in common law in 1791. a. at common law, separate courts of law and equity existed. Since the plaintiffs or defendant had a right to a jury trial in all action at law, but not actions in equity, the right of jury trial is preserved only in actions at law. b. although the distinction between law and equity jurisdiction was never absolute, the distinction has become less clear in modern times with the merger of law and equity and the creation of new claims in response to social and political order that is more complex and different from 1791. Key distinction between common law and equity courts is the remedy. If you see in the case that the P is seeking money damages, that will be correlated to the right of a jury trial, while if the P is seeking specific relief, they may not have the right to a jury trial. y Suits involving land, you have to ask whether the appropriate remedy would be $ damages or an equitable remedy. In post-merger practice, the right-of-jury trial problem arises in several recurring contexts:

93

1) where legal and equitable claims are presented in a single lawsuit by virtue of some joinder device; e.g. compulsory or permissive counterclaims 2) where a party asserts a single claim, but request several forms of remedy, some of which are equitable; (P files a breach of contract action against the D and seeks an accounting to determine how much the defendant owes her and a judgment for that amount) 3) where a party files a claim that was only available in equity and requests a legal remedy (the P files a shareholders derivative lawsuit, where the shareholders right to sue on behalf of the corporations is historically an equitable matter and the claim of the corporation against the defendants is a legal matter if it raises legal issues.) Rule now is if you only have 1 claim, you follow historical rule. 2 areas that have caused problems 1. What do you do for new Congressional right of actions? y try to figure out whether it would have historically been heard in a common law or equitable court. 2. What do you do when there is numerous claims? Say P has both a legal and equitable claim against D? y some states will look at the dominant claim FOR FEDERAL COURT The Sup. Court applies a general prudential rule (from Beacon Theatres) which seeks to identify the rights (or issues) involved and remedies sought in a merged lawsuit that can be characterized as legal, and then to assure that they are given a jury trial if a party so desires. As a practical matter, the major difference between the historical test and Beacon Theatres Test is that the general prudential rule requires a jury trial whenever legal and equitable matters are presented in the same lawsuit. y in federal court you dont go claim by claim, you go issue by issue, and all the issues that are legal in nature have to be tried first, or together with the equitable claims to a jury. Amoco Oils v. Torcomian Facts: Amoco (P) sued Ds in federal court to eject Ds from a gas station Ps owned and Ds had arranged to lease from Ps leasees who could no longer operate it. D's wanted to become one of Ps franchisees but the franchise agreement was never consummated. In addition to ejectment, P sued to enjoin Ds from operating the station and for damages for wrongful operation of the station. Ds counterclaimed for damages and specific performance and demanded a jury trial. ON the eve of trial, P abandoned portions of the complaint relating to damages and other claims that might be construed as

94

legal and, thereby foreclosed Ds right to a jury trial. Ds request for a jury trial was denied. After suffering an adverse judgment, Ds appeal. Issues: (1) Did the district court erroneously deny Ds demand for a jury trial as to any legal claims seeking legal relief in Ps complaint? (2) Did the district court erroneously deny Ds demand for a jury trial as to any legal claims seeking legal relief presented in a compulsory complaint? (3)If the right to a jury trial was erroneously denied to Ds, was such denial harmless error? Held: (i). yes (ii) yes (iii) No. Judgment vacated and remanded

y Issue is not whether it is mainly an equitable claim, it is whether there are any legal claims, and those claims must be tried first to a jury, then the rest of the equitable claims can be brought to a court to be heard. y Neither the joinder of an equitable claim with a legal claim nor the joinder of a prayer for equitable relief with a claim for legal relief as to a legal claim can defeat an otherwise valid 7th amendment right to a jury trial. y Under federal law, which controls in this diversity, actions seeking ejectment are deemed to be legal, not equitable. y Even an equitable main claim cannot preclude a jury trial on a legal counterclaim, at least when the counterclaim is compulsory. y Ds stated a compulsory counterclaim that included a claim for legal relief when they sought damages for breach of contract.

JUDGEMENT AS A MATTER OF LAW Suppose there is going to be a trial, what kind of rules are in place to control the jury. Burden of Persuasion: The burden of persuasion defines the extent to which a trier of fact must be convinced of some proposition in order to render a verdict for the party who bears it. In civil cases the burden of persuasion is lower than beyond a reasonable doubt, such as the preporderance of evidence standard. Burden of Production: the burden on production requires a party to produce, to find and present evidence in the first place. P/D will lose if they have the burden of production

95

and failed to satisfy the burden of production by coming forward with evidence from which a rational trier of fact could conclude some proposition of material fact. 3 situations where the case can be taken away from the jury. a) summary judgment (Hutchens, Celotex) RULE 56 b) judgment as a matter of law - directed verdict RULE 50(a) c) renewed judgment as a matter of law (JNOV) RULE 50(b) ** Cant make JNOV if you dont make motion for directed verdict during the trial, and also have to make JNOV in order to preserve issue of sufficiency of evidence on appeal** Judgment as a Matter of Law (Directed Verdict) : RULE 50 permits a party at the close of the other partys case to move for judgment as a matter of law, (j.m.l.). In so moving, the defendant would be asking the judge to take the case away from the jury to prevent it from considering the evidence and reaching a verdict. The ground for the motion would be that the evidence presented would only support one result: 50(a) says that there is no legally sufficient evidentiary basis for a reasonable jury to find for that party on that issue. y To grant such a motion results in final judgment being entered against the party with the burden that she has failed to carry. y A judge should direct a verdict only if there is no rational basis for a jury to find in favor of the party agiasnt whom the verdict is directed. y Same as the standard for summary judgment. Instructions and Comment:Rule 51- The judge frames the question by instruction the jury on the law. Instructions explain the substantive law that applies to the case, and the judge explains in a sequential way the decisions that a jury must reach in a given case. Judgment as a Matter of Law (Judgment Not Withstanding the Verdict JNOV): Essentially a late ruling on the earlier motion for a judgment as a matte of law. The grounds for a j.n.o.v. are identical to those for a preveridct judgment as a matter of law: That there is no legally sufficient evidentiary basis for a reasonably jury to find for the party against whom the motion is made. Rule 50(a). Only the timing of the motions is different, one coming before the verdict, the other coming after. New Trial: If the judge cannot say that there was no evidence to support the verdict, but thought one party was very strong and the other party was weak, and the weaker party won, then under RULE 59 the judge can grant a new trial on her own initiative (Rule 59(d)). Difference between a JNOV and a new trial is that JNOV reverses the verdict and says that the other party is the winner, while a new trial does not, standard is lower for a new trail. There are two reasons for granting a new trial. (a) Flawed procedures: New trial may be granted when the judge concludes that the process leading up to the verdict

96

ahs been flawed. (impermissible arguments, wrongful admittance of evidence, jury misbehavior). (b) Flawed verdicts: If the verdict is against the weight of the evidence than the judge, if no party motions for it, can grant a new trial. Conditional New Trials: 1. There can be a new trial just on damages; two aspects y remittitur:when Ds motion for excessive damages,the judge orders a new trial unless the plaintiff agrees to accept reduced damages.(allows P to recover reasonable range, of damages) y additur: where the judge orders a new trial unless the defense allows the damages to be increased. (not permitted in the federal system b/c you are granting something that the jury never decided) Say jury returns verdict for 10K, but judge thinks only a verdict between 4-6K would be reasonable.

Reid v. San Pedro Railroad Facts: Reid (P) sued D to recover damages for killing Ps cow by Ds train. The cow, which was in an enclosed field near the railroad tracks, apparently got onto the tracks either through a broken fence or an open gate. The cow was found about a mile from the broken fence and close to the open gate. The jury rendered a verdict in favor of P. D appeals. Issue: Was the evidence (which fails to show where and under what circumstances the cow got onto the tracks) sufficient to support the verdict. Held: No. Judgment reversed. y Under applicable statutory law, D could not be liable if the cow passed through the gate. Therefore, P could recover only if the cow passed through the broken fence, and P had to show this by sufficient evidence. This P fails to do. Comment: P failed to satisfy her burden of production because there was no rational basis for the jury to find in Ps favor based solely on the evidence P produced. The cow could have passed through either the broken fence or open gate. Notes: Page would argue that the burden of persuasion is not really the one at issue here, really burden of production because the P has offered no evidence on the issue of causation.

97

y two cars collide, and no one sees what happens. how is the case decided if one of noone knows which car crossed the center line? o no one will win because there is no way to produce the evidence, although now days the cars could be examined to figure out who caused the accident. Pennsylvania Railroad v. Chamberlain Facts: The decedent, a brakeman for the company, fell to his death from a string of railroad cars during a maneuver to break up and make up chains of cars. The administratrix claimed that the decedent fell from the train on which he was riding because of a collision caused by the company's negligence. The only witness for the administratrix testified that he heard a bump, but that it was not loud enough for him to turn around. He testified that he did not actually see a collision. Three employees, riding the nine-car string, testified that no such collision occurred. Issue: Was the evidence of Ds negligence legally sufficient to send the case to jury? Held: No, judgment reversed y P failed to satisfy the burden of production, because the third party could not possibly have seen the alleged collision due to where he was standing. The testimony was suspicious and insubstantial, and the crash that he heard could have been form anything since it was common on the railroad track. y Considering the weight of the testimony of Ds witnesses, it is clear that P couldnt have satisfied the burden of production. When the evidence tends equally to sustain either of 2 inconsistent propositions neither of them can be said to have been established by legitimate proof. A verdict in favor of the party bound to maintain one of those propositions against the other is necessarily wrong, and where the evidence is so overwhelmingly on one side as to leave no room to doubt what the fact is, the court should give a preemptory instruction to the jury. Notes: In the typical case where you have conflicting evidence, that is for the jury. It is only in the rare case, where a judge will decide on conflicting evidence, discrediting the testimony of one side. 1st Category: Reid/ Houchens are cases where there is NO evidence on one side, and it will not make it to the jury. 2nd Category in the Chamberlain/ Bias case there is evidence on both sides, and normally this would go to the jury, but in a rare instance the Court will discredit the testimony from one side. 3rd Category: No dispute on the facts, will get tried by a jury.

98

Only should grant a new trial if there is something seriously wrong with the case, and only grant a JNOV if there is something seriously wrong where no reasonable jury could have reached that result. Say P wins on a verdict: D wants 1. JNOV 2. New Trial 3. Verdict P wants 1. Verdict to stand 2. New Trial 3. JNOV 50 (c)(d) is saying that on appeal, if D is granted JNOV, then P should be able to argue for re-instatement of the verdict, or a new trial.

RES JUDICATA (Claim Preclusion) & COLLATERAL ESTOPPEL (Issue Preclusion)


Res judicataGoal is for parties not to fragment claims. Prevents a plaintiff from suing on a claim that already has been decided and also prevents a defendant from raising any new defense to defeat the enforcement of an earlier judgment. It also precludes relitigation of any issue, regardless of whether the second action is on the same claim as the first one, if that particular issue actually was contested and decided in the first action. A judgment in a prior suit between the same parties is final not only as to all matters that were in fact offered and received to sustain or defeat the claim but also as to all matters that might have been offered for that purpose. A party may not litigate a claim and then, upon an unsuccessful disposition, revive the same cause of action with a new theory.

Four Factors Are Considered in Determining the Validity of a Plea of Claim Preclusion: 1) Was the claim decided in the prior suit the same claim being presented in the action in question? 2) Was there a final judgment on the merits? (findings of fact, conclusions, binding judgment)

99

3) Was the party against whom the plea was asserted a party or in privity with a party to the prior suit? (Same parties or successors in interest) 4) Was the party against whom the plea was asserted given a fair opportunity to be heard on the issue? The two leading tests for determining when a claim has been split or what constitutes a different casue of action, are the traditional test (common core of operative facts test) and the modern test (same transaction test). The modern test has broader preclusive effect thatn the traditional test in that a lawsuit based on a transaction that gives rise to several legal theories, or primary rights, cannot yield a subsequent lawsuit under the transactional test, but could do so under the traditional test. Frier v. City of Vandalia Facts: Plaintiff had several cars towed by defendant for blocking access to a street. After losing in state court seeking replevin, plaintiff turned to the district court alleging a due process violation under U.S. Const. amend. XIV because defendant towed his cars without subsequently holding a prompt hearing. The court affirmed the district court's dismissal because plaintiff had his day in court in the replevin action, and therefore, these claims were barred by claim preclusion Issue: Was the section 1983 action the same cause of action as the replevin action? Held: Yes, judgment affirmed. y Under 28 U.S.C. section 1738 the law of Ill. governs the preclusive effect to be given to the judgment in the replevin action. (Look to the law of the state that rendered the judgment) y Ill. applies the traditional test for determining the identity of a cause of action. Causes of action are identical where the evidence necessary to sustain a second verdict would sustain the first, i.e. where the causes of action are based upon a common core of operational facts. y P has attacked the same primary right, the same core of operative fact, or the same conduct- towing and detaining cars without a determination of a parking violation in all his suits y Parties hould be required to consolidate all related matter in a single lawsuit. y Since they both rise out of the same occurrence, the towing of the cars, this should be enough to preclude the second cause of action under 1983. Comment: As the majority recognizes, Ill courts focus on the similarities between cause of action alleged in both suits (the traditional test), not on whether there is a common factual transaction (the modern test). The common operative facts required for claim preclusion are those facts necessary to sustain the cause of action, ie. the evidence.

100

Consistency: Res judicata was applied in Frier to a situation involving the actual assertion of a claim in a prior lawsuit and the attempt to reassert what in essence was the saem claim in a subsequent lawsuit.

I McDs

Martinos BK (ended w/ consent judgment)

II Martinos McDs for antitrust (trial court said it should have been asserted as a compulsory counterclaim in the first law suit)

Martino v. McDonalds System Facts: Plaintiff restaurateur entered into a franchise agreement with defendant fast food franchise. The contract provided that neither plaintiff nor a member of his immediate family would acquire a financial interest in a competing self-service food business without the written consent of defendant. However, plaintiff's son purchased an interest in a competing fast food franchise without defendant's consent and plaintiff financed the transaction. Defendant brought a breach of contract action against plaintiff. The lawsuit ended in a consent judgement. Issue: Does either 13(a) [A pleading shall state as a counterclaim any claim which at the time of serving the pleading the pleader has against any opposing party, if it arises out of the transaction or occurrence that is the subject matter of the opposing party's claim] or res judicata bar Ps antitrust claim? Held: Yes, judgment affirmed. y Failure to assert in a pleading a compulsory counterclaim that at the time of the pleading, the pleader has against the opposing party results in the loss of the counterclaim. Since Ps filed no pleading within the meaning of Rule 7(a) (e.g. and answer), Rule 7(a)s bar is inapplicable. y The well-settled rule for the purpose of determining the res judicata effect of a judgment is that cause of action comprises defenses, such as the alleged antitrust violation here, that were or might have been raised. Thus, if such defenses were not presented in their entirety in the first action, they are lost. This rule, however, does not apply to this case because, inasmuch as Ps did not n any way assert a defense or counterclaim in the first lawsuit, they assert a different claim or cause of action in the present lawsuit. Res judicata does not preclude litigation of a difffernt claim against the same party. y It held plaintiff's claim fell within the narrow class of common law compulsory counterclaims. Additionally, it held that plaintiff was barred from raising his antitrust claim because of the doctrine of res judicata.

101

Collateral EstoppelIn any lawsuit between two parties who have engaged in previous litigation, the first question will be whether this is the same claim. Only if one answers that question in the negative does the second question arise: are there then issues that the first case precludes from relitigation?

Prerequisites for Collateral Estoppel (1) The issue in the second case must be the same as the issue in the first. (2) The issue must have been actually litigated. (3) The issue must have been actually decided. (4) The issue must have been necessary to the court's judgment. y only applies when the second action is not barred by res judicata y applies only to issues that were actually determined in the prior action, NOT those that might have been determined and werent o if you cant tell what the basis for the judgment is, it could have been one of several thing, none of those could have the collateral estoppel effect o where two issues are actually decided in a case, and one of them is not a necessary ground for the result in the case, then it is not given collateral estoppel effect. o where the case has been decided on more than one ground, and you know what the decision on those grounds was, and either one of them independently can support the result than either  Restatement says that you dont give any collateral estoppel  Majority Rule- give all of them collateral estoppel effect IMPORTANT RULES y If there are 2 issues, are you cant tell which one the case is decided on, NEITHER gets collateral Estoppel Effect o If you have judgments that are both independently sufficient to support the judgment, neither should be given collateral estoppel effect.

Automobile Accident Hypos A sues B = Judgment B C sues B = C is not bound by this judgment, C is a separate entity

102

A v. B = Judgment A (B was negligent) C v. B = Should C be allowed to use the first judgment to preclude B from denying negligence? y C cannot take advantage of judgment for A, because B would not have been able to take advantage of a positive judgment = Mutuality of Estoppel

A v. B = A (not negligent) A v. C = Can A use first judgment to establish judgment for negligence in this action? --NO

A v. B = Judgment B (A was negligent) A v. C = Can C raise as a defense that A was negligent? recognizes mutuality of estoppel.

No, if thejurisdiction

y Defense Mutual Collateral Estoppel: The infringer can raise the successful defense in the second action. Use of collateral estoppel defensively (by a prior successful defendant), to prevent relitgation of an issue by a plaintiff who was a non-party to first action, was perceived as unfair because the non-party never had an opportunity to be heard in the first action. Thus, due process prevented application of defensive collateral estoppel because the non-party must be given an opportunity to be heard to have its day in court. o Rationale for erosion of the bar against defensive collateral estoppel. Defensive collateral estoppel is now recognized as a means of preventing a defendant from being harassed by serial litigation. If a defendant has ben sued on an issue and won, then the defendant ought to be able to assert its favorable judgment on that issue to prevent subsequent plaintiffs from suing on the identical issue. y Offensive Mutual Collateral Estoppel: The mutuality requiremtn prevented a noon-party to the first lawsuit from offensively asserting collateral estoppel, in a subsequent litigation, on an issue against a party to the first lawsuit. o Fairness concerns relative to the defendant:In abandoning the mutuality requirement for assertions of offensive collateral estoppel, the SC requires that the court in the second action determine whether the defendant will be unfairly prejudiced by the non-party plaintiffs offensive use of a perior plaintiffs favorable judgment against the defendant. Considerations include. y forseeability of the subsequent lawsuit y sideline y aberrational first judgment

103

1) Say trainwreck where 50 people are injured, and in the first action 1 of the injured parties sues the railroad and loses. The second sues, loses. The third P, is not bound by the first two suits. Say in the 10th case the person wins against the railroad. So say in the 11th judgment, can the P take advantage of judgment and get collateral estoppel effect? --- NO, because it seems like 10th is a fluke judgment. 1a) If the 1st person wins the suit, will the 2nd person get the advantage of the judgment? -- It shouldnt , because 1st suit could be an aberration.

104

S-ar putea să vă placă și